SlideShare ist ein Scribd-Unternehmen logo
1 von 80
Dear students, get ICFAI latest Solved assignments and case study
help by professionals.
Mail us at : help.mbaassignments@gmail.com
Call us at : 08263069601
AN INTRODUCTION TO DIFFERENTIATED LEARNING TOOLS
Participants in flexible learning programs have limitations on the nature of the
time they can spend on learning. Typically they are employed fully or partially,
pursuing higher studies or have other social and familial responsibilities.
Availability of time is a great constraint to these students.
To aidthe participants,we have developedfour unique learningtools as below:
Bullet Notes : Helps in introducing the important concepts in each unit
of curriculum, equip the
student during preparation of
examinations and
Case Studies : Illustrate the concepts through real life experiences

Workbook : Helps absorption of learning through questions based on reallife nuggets
PEP Notes :Sharing notes of practices and experiences in the Industry will help the student to
rightly perceive and get inspired to learn concepts at the cutting edge
application level.placementinterviews
Why are these needed?
 Adults learn differently from B. School or college going
students who spend long hours at campus.
 Enhancing analytical skills through application related learning
kits trigger experiential learning
 Availability of time is a challenge.
 Career success increasingly depends on continuous learning
and success
What makes it relevant?

How is it useful?


Where does this lead to?
As and when you get 5 to 10 minutes you can read one of these and absorb and comprehend.
Spending more time is your choice.
You can use the time in travel, waiting for meetings, lunch time, small breaks or at home
usefully.
Through these tools, the learning bytes are right sized for ease of learning for time challenged
participants.
The content starts from practice and connect to precept making it easy to connect to industry
and retain.
They can be connectedto continuous assessment process of the academic program.
Practitioners can use their real life knowledge and skill to enhance learning skills.
Immediate visualization of the practical dimension of the concept will offer a rich learning
experience.
Easier to move ahead in the learning process.

Will facilitate the student to complete the program earlier than
otherwise.Helpsstay motivated and connected.
When is it useful?

Principles of
Management
Workbook
© The ICFAI Foundationfor Higher Education (IFHE),
Hyderabad,December, 2014.All rights reserved
No part of this publication may be reproduced, stored in a retrieval system, used in a spreadsheet, or
transmitted in any form or by any means – electronic,mechanical, ph otocopying or otherwise – without
prior permission in writing from The ICFAI Foundation for Higher Education (IFHE), Hyderabad.
Ref. No. PM-WB-IFHE – 122014
For any clarification regarding this book, the students may please write to The ICFAI Foundation for
Higher Education (IFHE), Hyderabad giving the above reference number of this book specifying chapter
and page number.
While every possible care has been taken in type-setting and printing this book, The ICFAI Foundation
for Higher Education (IFHE), Hyderabad welcomes suggestions from students for improvement in future
editions.
Our E-mail ID: cwfeedback@icfaiuniversity.in
INTRODUCTION
Participants in ICFAI University Programs are eager to learn more from practice. They realize
that application orientation can enhance their learning and subsequent usage of management
precepts and practices. Picking out the principle behind real world events is critical to this
learning, as also identifying the alternative/solution using the principle. Towards this end the
institution has reengineered the Workbook.
The Workbook is a set of questions which typically illustrate a real life context from
contemporary corporate happenings and then poses a question to the student for reflection. The
narration of question helps the reader to reinforce the concept and facilitates the student to
enhance his/her capabilities in analyzing and interpreting the conceptual frameworks.
The examples depicting the names of existing persons or companies are taken from news
clippings/ published articles from various public domain websites or website of respective
companies. Since live examples reinforce the understanding of the students, the possible
responses are connected to the concepts taught directly or indirectly. In many cases the
alternatives provided are choices in a work situation based on alternative approaches.
These questions provoke the learner to start thinking from the application side and connect to
the knowledge that he will use to solve. Practitioners can connect better thereby improving the
learning experience. This form of assessment improves learning while assessing whereas the
conventional form is more about assessment of learning.
The learning outcomes expected are:
The examples are linked back to application of theoretical knowledge in the illustrated
real-time situation. This facilitates the student to develop analytical approach in similar
or related situations.
Application based approach which enhances absorption and retention significantly.
Exposure to the current incidences and situations in relation to important concepts of
the subject.
The Workbook format is also used for Assessment.
DETAILED CURRICULUM
UNIT 1: MANAGEMENT: AN OVERVIEW
Definition of Management- The Role of Management - Functions of Managers: Planning,
Organizing, Staffing, Leading, Controlling - Levels of Management: Top-level Managers,
Middle-level Managers, First-level Managers; Time spent in carrying out Managerial Functions
ᜀ Ā ᜀ Ā ᜀ Ā ᜀ Ā
anagement Skills and Organizational Hierarchy: Technical Skills, Human Skills, Conceptual
and Design Skills - Approaches to Management
UNIT 2: EVOLUTION OF MANAGEMENT THOUGHT
Early Approaches to Management: Robert Owen: Human Resource Management Pioneer;
Charles Babbage: Investor and Management Scientist; Andrew Ure and Charles Dupin:
Management Education Pioneers; Henry Robinson Towne (1844-1924) - Classical Approach:
Scientific Management-Frederick Winslow Taylor, Frank and Lillian Gilbreth, Henry Laurence
Gantt, Limitations of Scientific Management; Administrative Theory-Henri Fayol, Bureaucratic
Management, Limitations of bureaucratic management and administrative theory - Behavioral
Approach: Mary Parker Follet: Focusing on Group Influences Elton Mayo : Focusing on
Human Relations-Illumination experiments, Relay assembly test room experiments, Interview
Phase, Bank wiring observation room experiments, contributions and criticism of Hawthorne
studies Abraham Maslow: Focusing on Human Needs Douglas McGregor: Challenging
Traditional Assumptions about Employees Chris Argyris: Matching Human and Organizational
Development, Model I and Model II organizations - Quantitative Approach: Management
Science, Operations Management, Management Information Systems - Modern Approaches to
Management: Systems Theory, Contingency Theory, Theory Z
UNIT 3: SOCIAL AND ETHICALRESPONSIBILITIES OF MANAGEMENT
Social Responsibilities of Management -Arguments for and against Social Responsibilities of
Business: Arguments for Social Responsibilities of Business: Change in public expectations,
Business is a part of society, Avoiding intervention by government, Balance of responsibility
and power, Impact of internal activities of the organization on the external environment,
Protecting shareholder interests, New avenues to create profits, favorable public image,
Endeavor to find new solutions, Best use of resources of a business, Prevention is better than
cure - Arguments against Social Responsibilities of Business: Opposes the principle of profit
maximization, Excessive costs, Weakened international balance of payments, Increase in the
firm’s power and influence, Lack of necessary skills among business people, Lack of
accountability to society, Lack of consensus on social involvement -Social Stakeholders:
Shareholders, Employees, Customers, Creditors and Suppliers, Society, Government -
Measuring Social Responsiveness: Contributions, Fund-raising, Volunteerism, Recycling,
Valuing diversity, direct corporate investment, Quality of work life, Attention to consumers,
Pollution control, Social audits - Managerial Ethics - Types of Managerial Ethics: Moral
management, Amoral management, Immoral management.
Factors that influence Ethical Behavior: stages of moral development, individual
characteristics, Structural variables, Organization’s culture, Issue intensity - Ethical Guidelines
for Managers: Obeying the law, Tell the truth, Uphold human dignity, Adhere to the golden
rule, Primum non-nocere, Allow room for participation, Always act when you have
responsibility - Mechanism for Ethical Management: Code of ethics, Ethics committee, Ethics
audits, Ethics training, Ethics hotline.
UNIT 4: FUNDAMENTALS OF PLANNING
Definitions of Planning - Nature of Planning -Significance of Planning: Focuses Attention on
Objectives, Offsets Uncertainty and Risk, Provides Sense of Direction, Increases
Organizational Effectiveness, Provides Efficiency in Operations, Ensures Better Coordination,
Facilitates Control, Encourages Innovation and Creativity, Facilitates Delegation - Types of
Plans: Plans based on Organizational Level: Strategic plans, Tactical plans, Operational plans,
Plans based on Frequency of Use: Single-use plans, Standing plans, Plans based on their Time-
Frame: Short-term plans, Intermediate-term plans, Long-term plans - Steps in the Planning
Process: Analyzing Opportunities, Establishing Objectives, Determining Planning Premises,
Identifying Alternatives, Evaluating Available Alternatives, Selecting the Most Appropriate
Alternative, Implementing the Plan, Reviewing the Plan - Prerequisites of Effective Planning:
Establishing the Right Climate for Planning, Clear and Specific Objectives, Planning Premises,
Initiative at the Top Level, Participation in Planning Process, Communication of Planning
Process, Integrating Long-Term and Short-Term Plans, An Open Systems Approach,
Management Information System - Limitations of Planning: Lack of Accurate Information,
Time consuming Process, Expensive, Inflexibility, Resistance to Change, Environmental
Constraints, Lack of Ability and Commitment, False Sense of Security, Reluctance to Establish
Goals
UNIT 5: MANAGEMENT BYOBJECTIVES
Nature of Objectives: Hierarchy of Objectives, The Process of Forming Objectives and
Organizational Hierarchy, A Network of Objectives, Multiplicity of Objectives - Concepts of
MBO: Early Impetus to MBO, Emphasis on Performance Appraisal, Inclusion of Long-term
Planning in the MBO Process, The Systems Approach to MBO - The Process of MBO:
Developing overall organizational goals, Establishing specific goals for various departments,
subunits and individuals, Formulating action plans, Implementing and maintaining self-control,
Periodic Review, Performance appraisal - Benefits of MBO: Better Management, Clarity in
Organizational Action, Encouragement of Personal Commitment, Personnel Satisfaction, Basis
for Organizational Change, Development of Effective Controls - Limitations of MBO: Failure
to Teach MBO Philosophy, Failure to Give Guidelines to Goal Setters, Difficulty in Goad-
setting, Emphasis on Short-term Goals, Inflexibility, Other Dangers - Making MBO Effective:
Top Management Support, Training for MBO, Formulating Clear Objectives, Effective
Feedback, Encouraging Participation
UNIT 6: STRATEGIES, POLICIES AND PLANNING PREMISES
Nature and Purpose of Strategies and Policies: The Key Function, The Guide, The Need for
Operational Planning, The Effect on all Areas of Management - The Three Levels of Strategy:
Corporate-Level Strategy : The values-based approach, The corporate portfolio approach, The
BCG Matrix; Business-Level Strategy, Functional-Level Strategy - Strategic Planning:
Characteristics of Strategic Planning, Significance of Strategic Planning, Limitations of
Strategic Planning - Strategic Planning Process: Defining the Mission of the Organization,
Drawing up Organizational Objectives, Assessing Organizational Resources, Risks and
Opportunities, Formulating Strategy, Implementing Strategy, Monitoring and Adapting
Strategic Plans; Strategic Planning Vs. Operational Planning - Competitive Analysis in Strategy
Formulation: Environmental Assessment-Porter’s five competitive forces model,
Organizational Assessment-SWOT Analysis - Major Kinds of Strategies and Policies: Growth,
Finance, Organization, Personnel, Public relations, marketing - Porter’s Competitive Strategies:
Overall Cost Leadership, Differentiation, Focus - Strategy Implementation: Carrying Out
Strategic Plans – Technology, Human resources, Reward sys tems , Decision processes,
Structure; Maintaining Strategic Control - Effective Implementation of Strategy:
Communicating Strategies to all Key Decision-making Managers, Developing and
Communicating Planning Premises, Developing an Appropriate Fit between Organizational
Structure and Planning Needs, Ensuring that Action Plans Contribute to and Reflect Major
Objectives and Strategies, Developing Contingency Strategies and Programs, Reviewing
Strategies Regularly, Continuing to Emphasize Planning and Implementing Strategy, Creating a
Proper Organizational Climate - Planning Premises: Planning Premises Vs. Future
Expectations, Effective Premising – Selection of premises that affect the plans of the
organization, Development of alternative premises for contingency planning, Verification of
consistency of premises, Communication of the premises
UNIT 7: MANAGERIAL DECISION MAKING
Significance and Limitations of Rational Decision-making - Managers as Decision-makers: The
Rational Model, Non-Rational Models – Satisficing model, Incr emental model, Garbage-can
model - Decision-making Process: Identifying the Problem, Identifying Resources and
Constraints, Generating Alternative Solutions, Evaluating Alternatives, Selecting an
Alternative, Implementing the Decision, Monitoring the Decision - Types of Managerial
Decisions: Programmed Decisions, Non-programmed Decisions - Decision-making under
Certainty, Risk and Uncertainty: Decision-making under Certainty, Decision-making under
Risk, Decision-making under Uncertainty, Risk analysis, Preference or utility theory - The
Systems Approach to Decision-making: Management Information System, Decision Support
System - Group Decision-making: Forms of Group Decision-making – Interacting groups,
Delphi groups, Nominal groups
-Decision-making Techniques: Marginal Analysis, Financial Analysis, Ratio Analysis, Break-
even Analysis, Operation Research Techniques – Linear Pr ogramming, Queuing or waiting-
line method, Game theory, Simulation, Decision-tree
UNIT 8: FUNDAMENTALS OF ORGANIZING
Definitions of Organizing - Benefits of Organizing - Traditional Perspectives on Organizing:
Four principles from Fayol and Taylor’s theories of organizing: Unity of command, Well-
defined hierarchy of authority, Authority at par with responsibility, Downward delegation of
Authority and not Responsibility; Challenges to the Traditional View of Organizations –
Bottom-up authority, Environmental complexity and unce-rtainty - Closed System Vs. Open
System: Closed System view of Organizations, Open System view of Organizations – Daniel
Katz and Robert Kahn’s Open-system characteristics, Developing an open system model -
Formal Vs. Informal Organization - The Process of Organization: The Logic of Organizing,
Some Misconceptions - Bases for Departmentation: Departmentation by Simple Numbers, by
Time, by Process or Equipment - Choosing the Pattern of Departmentation.
UNIT 9: STRATEGIC ORGANIZATION DESIGN
Span of Management: Tall versus Flat Structure, Factors Determining an Effective Span – Trained
subordinates, Clarity of delegation of authority, Clarity of plans, Use of objective standards, Rate of
change, Communication techniques, Amount of personal contact needed, Organizational levels, Use
of staff assistance, Supervision by others, Other factors - Authority Defined - Power:Bases of
Power:Legitimate Power, Expert Power, Referent Power, Reward Power, Coercive Power - Line
and Staff Relationships: Concept of Line and Staff, Functional Authority, Line and Staff Conflicts –
Viewpoint of line m anagers, Viewpoint of staff managers, Nature of Line and Staff Relationship,
Avoidance of Line and Staff Conflict - Centralization Versus Decentralization: Factors responsible
for relative centralization and decentralization in an organization: History of an Organization,
Availability of Competent Managers, Size of the Organization, Geographical Dispersion,
Complexity of Tasks, Time Frame of Decisions, Importance of a Decision, Planning and Control
Procedures, View of Subordinates, Environmental Influences - Delegation of Authority: Factors
Affecting Delegation of Authority
– The delegator’s aspect,The delegant’s aspect,The orga nizational aspect - Balance:The Key
to Decentralization
UNIT 10: STRATEGIC ORGANIZATION STRUCTURE
Ensuring Understanding of Organizational Structure: Teaching the Nature of the Organizations’
Structure, recognizing the Importance of the Informal Organization and the Grapevine -
Designing Organizational Structures: An Overview: Which Comes First: Strategy or Structure?,
Factors Influencing Organization Design - Major Structural Alternatives: Advantages,
Disadvantages and Uses of Functional Structure, Divisional Structure – Product divisions,
Geographic divisions, Customer divisions, Hybrid Structure, Matrix Structure - Strategic
Business Units
UNIT 11: EFFECTIVE ORGANIZING AND ORGANIZATIONAL CULTURE
Prerequisites for Effective Organizing - Avoiding Mistakes in Organizing by Planning:
Planning for the Ideal, Modification for Human Factor, Advantages of Organization Planning -
Avoiding Organizational Inflexibility: Signs of Inflexibility, Avoiding Inflexibility through
Reorganization, The Need for Readjustment and Changes - Avoiding Conflict by Clarification:
Organization Charts, Position Descriptions -Organizational Culture: Meaning, Significance and
Characteristics of Organizational Culture, The organizational socialization process and steps -
organizational Environment for Entrepreneuring and Intrapreneuring
UNIT 12: HUMAN RESOURCE MANAGEMENTAND STAFFING
Human Resource Management: An Overview :Human Resource Planning, Forecasting
manpower demand, Forecasting manpower supply-Internal labor supply and External labor
supply, Human resource actions, Staffing , Training and Development, Performance Appraisal,
Compensation - Recruitment: Sources of Recruitment, The Recruitment Process - Selection:
The Selection Process - Socialization Process of New Employees.
UNIT 13: PERFORMANCEAPPRAISAL ANDCAREER STRATEGY
Significance of Appraisal - Informal vs. Formal Appraisals - Performance Rating Methods:
Graphic Rating Method, Behaviourally Anchored Rating Scale (BARS) - Criteria for
Appraising Managers: Appraising Manages against Verifiable Objectives, Appraising
Managers as Managers - Formulating Career Strategy: Preparation of a Personal Profile,
Development of Long-range Personal and Professional Goals, Analysis of Environment Threats
and Opportunities, Analysis of Personal Strengths and Weaknesses, Development of Strategic
Career Alternative, Consistency Testing and Strategic Choices, Development of Short-range
Career Objectives and Action Plans, Development of Contingency Plans, Implementation of a
Career Plan, Monitoring Progress
UNIT 14: ORGANIZATIONAL CHANGE AND ORGANIZATIONAL
DEVELOPMENT
Organizational Change: Factors that Lead to Organizational Change, Sources of Resistance to
Change – Individual and Organizational resistance, Change P rocess - Planned Change through
Organization Development: The objectives of OD - Organizational Development Process:
Diagnosis, Intervention – Process consultation, Team building , Third-party intervention,
Survey Feedback, Technostructural activities, Skill development, Organizational culture
change; Evaluation - Approaches to Manager Development: On-the-Job Training, Off-the-Job
Training - Organizational Conflict: Sources of Conflict, Managing Conflict
UNIT 15: MANAGING AND THEHUMANFACTOR
The Nature of People: Individual Difference, The Importance of Personal Dignity, Considering
the Whole Person, Multiplicity of Roles - Behavioral Models: Edgar H. Schein’s Model of the
Complex Person, Lyman Porter’s Model of Human Nature, McGregor’s Theory X and Theory
Y, Three Managerial Models by Raymon E. Miles -Managerial Creativity: The Creative
Process, Techniques to enhance Creativity; The Creative Manager
UNIT 16: MOTIVATING EMPLOYEES FORJOB PERFORMANCE
Definitions and Meaning of Motivation - Classification of Motivation Theories: Content
Theories of Motivation – Maslow’s needs hierarchy theory, Herzberg’s two-factor theory,
McClelland’s needs theory, Alderfer’s ERG theory; Process Theories of Motivation – Vroom’s
expectancy theory, The Porter-Lawler model, Equity theory of J Stacy -Motivational
Techniques: Rewards, Participation, Quality of Work Life (QWL), Job Enrichment - A Systems
and Contingency Approach to Motivation
UNIT 17: LEADERSHIP
Definition and Meaning of Leadership -Key Elements of Leadership -Leadership Theories:
Trait Theory of Leadership, Behavioral Theories, Likert’s Four Systems of Management, The
Managerial Grid, Situational or Contingency Theories, Transformational Leadership Theory
UNIT 18: MANAGING COMUNICATIONS
Definitions of Communication - Significance of Communication in Organizations -
Communication Process -Communication Flows in an Organization: Downward, Upward,
Crosswise Communication - Barriers to Communication: Lack of Planning, Badly Expressed
Messages, Faulty Translations, Unclarified Assumptions, Semantic Distortion, Loss by
Transmission and Poor Retention, Communication Barriers in the International Environment,
Inattention and Premature Evaluation, Impersonal Communication, Insufficient Adjustment
Period, Information Overload, Lack of Trust in the Communicator, Other Communication
Barriers - Gateways to Effective Communication: Interpersonal Trust, Effective Listening,
Proper Feedback, Non-verbal Cues, Non-directive Counseling
UNIT 19: THE CONTROLFUNCTION
Planning and Controlling - Importance of Controlling: Coping with Uncertainty, Detecting
Irregularities, Identifying Opportunities, Handling Complex Situations, Decentralizing
Authority, Minimizing Costs - Levels of Control: Strategic Control, Tactical Control,
Operational Control - Basic Control Process: Determining Areas to Control, Establishing
Standards, Measuring Performance, Comparing Performance against Standards, Recognizing
Good or Positive Performance, Taking Corrective Action when Necessary, Adjusting Standards
and Measures when Necessary - Direct Control vs Preventive Control - Types of Control:
Control Based on Timing, Cybernetic and Non-cybernetic control - Requirements for Effective
Controls - The Certified Management Audit and Enterprise Self-Audit
UNIT 20: CONTROL TECHNIQUES
Major Control Systems: Managerial level – Types of cont rol system; Nature of timing – Types
of control system - Financial Control: Financial Statements, Ratio Analysis - Budgetary
Control: Responsibility Centers, Standard cost centers, Discretionary expense centers, Revenue
centers Profit centers, Investment centers, Uses of responsibility centers - Quality Control:
Quality Circles, Total Quality Management (TQM) - Inventory Control
UNIT 21: PRODUCTIVITYAND OPERATIONS MANAGEMENT
Production and Productivity - Productivity Problems and Measurement - Operations Research,
Production & Operations Management: Operations Management and its Importance,
Operations Research for Planning, Controlling and Improving Productivity, The essentials of
operations research, operations research methodology - Some Operations Research Techniques:
Linear Programming, Inventory Control, Significance of Inventory, Costs of Inventory,
Economic Order Quantity, Just-in-time inventory system, Kanban, Distribution logistics, Time-
event Networks, Value Engineering ,Work Simplification - Limitations of Operations Research:
Magnitude of Computation, Gap between Managers and Operations Research, Lack of
Quantification and Involvement of Qualitative Factors
UNIT 22: MANAGEMENTINFORMATIONSYSTEMS
Management Information: Meaning Attributes and Information needs of Managers -
Components of an Information System: Hardware, Software, People, Data, Procedures - Types
of Information Systems: Transaction Processing Systems, Office Automation Systems,
Decision Support Systems, Executive Support Systems - Management Information Systems:
Evolution of MIS, Computers and MIS, Advantages of MIS, Difference between MIS and DSS.
CONTENTS
Multiple Choice Questions 1
Unit 1 – Management: An Overview 1
Unit 2 – Evolution of Management Thought 3
Unit 3 – Social and Ethical Responsibilities of Management 5
Unit 4 – Fundamentals of Planning 7
Unit 5 – Management by Objectives 9
Unit 6 – Strategies, Policies and Planning Premises 11
Unit 7 – Managerial Decision Making 13
Unit 8 – Fundamentals of Organizing 15
Unit 9 – Strategic Organization Design 17
Unit 10 – Strategic Organization Structure 19
Unit 11 – Effective Organizing and Organizational Culture 21
Unit 12 – Human Resource Management and Staffing 23
Unit 13 – Performance Appraisaland Career Strategy 25
Unit 14 – Organizational Change and Organization Developmen t 27
Unit 15 – Managing and the Human Factor 29
Unit 16 – Motivating Employees for Job Performance 31
Unit 17 – Leadership 33
Unit 18 – Managing Communications 35
Unit 19 – The Control Function 37
Unit 20 – Control Techniques 39
Unit 21 – Productivity and Operations Management 41
Unit 22 – Managerment Information Systems 43
Multiple Choice – Answers and Explanations 45
MULTIPLE CHOICE QUESTIONS
UNIT 1 – MANAGEMENT: AN OVERVIEW
In a management quiz, the quiz master asked as to who has defined management as “the process
of designing and maintaining an environment in which individuals, working together in groups,
efficiently accomplish selected aims.” The correct ans wer is__________________.
a. Harold Koontz b. Heinz Weihrich
c. Henry Mintzberz d. a & b
e. a & c
On observing a drop in its first quarter earnings, a manager in a mobile manufacturing industry
wanted employees to pool ideas for innovation. The employees came up with an idea of
developing new apps in mobile where it had to get the permission from government officials to
import some of the chips from a foreign company. Then the manager assured themto get the same
talking to government officials. Name the decisional roles played by the manager.
a. Entrepreneurial role b. Disturbance handler
c. Resource allocator d. a & c
e. a & b
Ritesh had a plan to start cab facility dealing with wide range of cars from normal to luxuries. His
objective was to provide the cheapest cab services with maximum comfort for the passengers. He
then selected the cities to locate offices and recruited people to manage those offices. He then
delegated the duties and responsibilities to the staff and took charge of supervising the overall
branches. What are the different functions Ritesh has performed?
a. Planning &Organizing b. Staffing
c. Leading d. Controlling
e. All the above
Karthik as a supervisor has to manage the day to day tasks in a manufacturing division. He also
has to check the performance of the staff and ensure implementation of operational plans
according to the organizational requirements. Identify the managerial level that Karthik fits into.
a. Top-level managers b. Middle-level managers
c. First-level managers d. Second-level managers
e. None of the above
The top-level managers spend time in planning and organizing and first-level supervisors spend
time in communication. In addition to these there are many factors. Name the factor which is
similar at all the hierarchical levels in management.
a. Controlling b. Supervising
c. Operating d. All the above
e. None of the above
Karan is Head-Accounts manager in a construction company. He co-ordinates with different site
supervisors and accumulates all the details of the accounts from different construction sites and
branches periodically, consolidates and reports the details to his superior. He prepares the balance
sheet and gets it audited by internal and external auditors as well. What are the different skills that
are demonstrated by Karan?
1
Principles of Management
a. Technical skills b. Human skills
c. Conceptual and Design skills d. a & b
e. a & c
Manisha is a team leader in a software company. In order to meet her project deadlines, she
conducts meetings regularly, checks the progress status and motivates her team members
accordingly. She also helps them to tackle the problems encountered in the process. Identify the
skill exhibited by her to get her work done.
a. Technical skills b. Human skills
c. Conceptual skills d. Design skills
e. All the above
A marketing company proposed to combine its marketing and sales department to foster
coordination and improve sales. This decision has changed the job profiles of all the sales and
marketing team and the HR manager had to communicate their new profiles. Accordingly,
changes were made in the organizational structure and hierarchy in the marketing department.
Identify the management approach applied in the case.
a. The Interpersonal Behavior Approach b. The Empirical or Case Approach
c. The Operational Approach d. The McKinsey’s 7-S Framework
e. The Mathematical or “Management Science” approach
When a supermarket found that long queues are obstructing the billing staff to concentrate, it has
planned to increase billing stations to reduce the crowd. The management collected information
on number of customers arriving at each station at each branch and used statistical methods to
come out with number of billing stations to be increased in each branch. Name the management
approach used in the case in decision making.
a. The Interpersonal Behavior Approach b. The Management Science Approach
c. The Cooperative Social Systems Approach d. The Decision Theory Approach
e. None of the above
A Supervisor observed that an employee was coming late even after several reminders and as per
the company’s rule he needs to be dismissed. But on enquiry, the manager found that his mother
was seriously ill. He understood the circumstances and counseled him to make alternate
arrangements. Find out the management approach that manager has used.
a. The Contingency or situational Approach b. The Operational Approach
c. The Socio-technical Systems Approach d. The Empirical or Case Approach
None of the above
2
Multiple Choice Questions
UNIT 2 – EVOLUTION OF MANAGEMENT THOUGHT
Management students in a class were being taught about the evolution of management. A student
in the class raised doubt as to who was the first to contribute to the study of management concept.
Identify the person fromthe following:
a. Charles Babbage b. Robert Owen
c. Henry Robinson Towne d. Yale and Towne
e. Andrew Ure
In a quiz competition at a Management School, Quiz master asked to name the person who is
widely known as the ‘Father of modern computing” and also a pi oneer in the field of
management. He is the person who invented the world’s fir st mechanical calculator and the
“analytical engine”. The student’s answer was
a. Henry Laurence Gantt b. Frank and Lillian Gilbreth
c. Andrew Ure and Charles Dupin d. Robert Owen
e. Charles Babbage
Harry was sharing his job experiences with one of his friends and was explaining how their
management deals with every issue in a scientific manner and provides the required support to the
employees. He was reminded of F.W. Taylor who proposed the scientific approach for the first
time. Immediately his friend argued that the first thought of scientific management was not from
Taylor and was by someone who inspired Taylor. Identify the person from the following.
a. Henry Robinson Towne b. Robert Owen
c. Charles Babbage d. Mary
e. None of the above
A supervisor in a company doubtful of completing the production s chedule on time wanted to
plan a strategy that could assure completion. He then decided to check and reduce the time taken
to complete each task without compromising with quality to set a standard. He also planned to
reward the worker who meets the established standards of performance set by the management.
Name the school of thought that inspired him
a. Scientific Management b. Administrative theory
c. Bureaucratic Management d. Quantitative approach
e. All the above
In an Operation Management classroom, the professor was explaining about the latest Program
Evaluation and Review Techniques (PERT) that is used as production control technique. A
student in the classroom had a doubt about the earlier techniques of control before the PERT
model was introduced. Pick up the answer from the following.
a. Micromotion study b. Time-and-motion study
c. Gantt Chart d. Piece-rate incentive system
e. Gantt bars
A parent interested to check their children’s progress, went to their school one day. She had to
comply with all the rules and regulations of the school to meet the concerned authorities and there
was a line of hierarchy before she met the principal to clarify her doubts. Name the kind of
administration followed by the school.
a. Scientific management b. Bureaucratic management
c. Social management d. Administrative management
None of the above
3
Principles of Management
Today, organizations are searching for multitasking individuals to meet the challenges of current
scenario and to encourage employees perform a variety of jobs in organization. There also exists a
conventional theory that strongly supports this modern practice. Name the person who proposed
the conventional theory from the following:
Mary Parker Follet: Focusing on Group influences
Elton Mayo: Focusing on Human Relations
Abraham Maslow: Focusing on Human Needs
Douglas McGregor
Chris Argyris: Matching Human and Organizational Development
An organization in its problem diagnosis found that its current manual data management system
was not effective and need an effective one. They searched for the latest technology and
implemented it where data collection, analysis and reports were easily accessible and manageable
to the managers. What is that management technique we are talking about?
a. Management Information Systems b. Operations Management
c. Management Science d. Scientific Management
e. Management science theory
Consequent to increased employee turnover, an organization decided to provide job security and
welfare measures for its employees. In addition, to reduce their boredom and to offer promotions,
training sessions were planned. This has reduced the labor turnover and absenteeism and
increased productivity. Name the theory applied in the case.
a. Theory Y b. Systems theory
c. Administrative theory d. Theory Z
e. Contingency theory
Gautam, a general manager in a manufacturing company tried to implement a strategy that was
successful in solving a problem. He tried to implement the same strategy to solve a similar
problem in another department but failed. From this experience he has learnt that there is no
single principle to manage all the situations in an organization. Extract the important theory that
he has learnt in the case from the following.
a. Systems Theory b. Contingency Theory
c. Theory A d. Theory Z
Theory X
4
Multiple Choice Questions
UNIT 3 – SOCIAL AND ETHICAL RESPONSIBILITIES OF MANA GEMENT
The K. C. Mahindra Education Trust, a part of Mahindra & Mahindra started a number of
education plans to change the lives of worthy students. More than 7.5 crore were sanctioned in the
form of grants, loans and scholarships to encourage education in India. It has about 3,300 children
in its project called ‘The Nanhi Kali’. To reach the n eglected children in rural areas, it has plans
to increase the number to 10,000 in the next two years. Name the responsibility it is taking up.
a. Political responsibility b. Social responsibility
c. Management responsibility d. Organization responsibility
e. Economical responsibility
Vodafone, as a part of its corporate social responsibility is trying to improve the efficiency in
energy consumption and to recycle its network equipment waste by 95%. Thus, it guarantees to
reduce their carbon dioxide emissions by 2020. In what way does this contribute to society?
a. Avoiding intervention by government b. Protecting shareholder interests
c. New avenues to create profits d. Balance of responsibility and power
e. None of the above
Haagen-Dazs is an ice cream brand from New York. A micro-blog was created by this company
to enlighten tweeters about the significance of honeybees.Through its “save the honeybee”
campaign, it created awareness on the consequences ofreducing bee population.It informed that
decline in bee population could reduce the global production and disappearance of several plant
species.It also alerted that when bees die out, Haagen-Dazs may just go out of business.What
kind of social responsibility is this?
a. Endeavor to find new solutions b. Best use of resources of a business
c. Favorable public image d. Balance of responsibility and power
e. Change in public expectations
McDonald’s sets its own standards on how suppliers have to take care of its employees. It also
started a program called “Flagship Farms Initiative (FFI)” throug h which it promotes the creative
and ethical farming practices. All these depict the social responsible practices of McDonald’s in
its supply chain strategy. Suppliers being one among the six major interest groups for
organizations, what are the other groups interested?
a. Shareholders b. Employees
c. Government d. Society
e. All the above
Infosys Technologies has a CSR foundation. It conducts training programmes and promotes
computer education in village schools. Name the category of social responsiveness Infosys has
demonstrated.
a. Fund-raising b. Recycling
c. Direct corporate investment d. Contributions
e. Quality of work life
Hindustan Zinc Limited reviews its monitoring systems for environmental safety at its mining
units. It maintains eco-friendly environment by building control plants for dust emission, mine
dumps for solid waste disposal, waste water treatment systems and more. Which category of
social responsiveness are we talking about?
a. Contributions b. Recycling
c. Attention to consumers d. Pollution control
Valuing diversity
5
Principles of Management
Apple Inc. is the largest US-based technology firmknown for its innovative products. At the same
time, it is also known for maintaining secrecy to an extent that secrecy has become one of the key
fundamentals of its corporate culture. In 2009, it was blamed for not disclosing the crucial
information regarding CEO Steve Job’s health to its shareholders. It also had to face criticism for
maintaining the information as a secret. This depicts the organization’s ____________.
a. Social responsibility b. Social audits
c. Code of conduct d. Social adaptability
e. Managerial ethics
Exxon Mobil’s main products are petroleum and gasoline owning refineries to drill and pump
crude oil all across the world. In 2007, Exxon Mobil was blamed for misguiding the public
claiming that it was concerned about the global warming and taking measures to reduce their
carbon footprint through commercial releases. They have removed these commercials
immediately when questions arose from the advertisements. What is the factor that influenced the
ethical behavior in the case?
a. Structural variable b. Issue intensity
c. Organization’s culture d. Individualcharacteristics
e. None of the above
Clinical trials are essential for the development of medical science and advancement in therapeutics. But
protecting the human rights and human safety plays a primary role in clinical trials. In India, the
regulatory agency that oversees and approves clinical trials is the CDSCO (Central Drugs
Standard Control Organization). Who is responsible for overseeing their activities?
a. Ethics Committee b. Ethics audits
c. Code of ethics d. Ethics training
e. None of the above
Adidas always looks out for innovative ways to connect directly with its suppliers/ workers. In 2012, the
Adidas group wanted to test mobile phone communication at one of the company’s main footwear
suppliers in Indonesia and started sending SMS to its workers. Through which channel, is Adidas
trying to get connected with its suppliers?
a. Ethics training b. Ethics hotline
c. Ethics committee d. Ethics audit
None of the above
6
Multiple Choice Questions
UNIT 4 – FUNDAMENTALS OF PLANNING
The CEO and Board members of a Food Products company set goals to increase its sales by
implementing new flavors of chips from the next quarter. Based on these goals, the middle level
managers started collecting the customer tastes through market research team and subsequently
decided flavors with the help of top level executives. Instructions were given to the operating
supervisors to make necessary arrangement for the production of new flavored chips. Which
nature of planning is depicted in this case?
a. Planning is goal-oriented b. Planning is all-pervasive
c. Planning is forward-looking d. Planning is an integrated process
All the above
Amit had a plan to supply building materials and was searching for the right place. He located a place
where a group of real estate ventures were approved by the government. He chose to start his
business there anticipating huge demand for the material supplies when the ventures get started.
Where can we fit the situation in significance of planning?
Planning provides sense of direction
Planning increases organizational effectiveness
Planning provides efficiency in operations
Planning offsets uncertainty and risk
None of the above
An FMCG organization’s goal was to exceed the sales target of the last quarter. Accordingly a meeting
was conducted with all the department heads. The marketing manager was assigned the
responsibility to ensure timely supply of goods to the sales executives coordinating with the
production manager. In turn production manager was required to produce goods on time
coordinating with finance department. This case depicts the significance of planning in_______.
a. Facilitating delegation b. Ensuring bettercoordination
c. Encouraging innovation and creativity d. Facilitating control
e. Providing efficiency in operations
To gain competitive advantage through quality products, top management of an Electric appliance
organization decided to implement Total Quality Management in the entire organizational
operations and processes within next five years. The Board of directors, CEO and heads of the
departments passed a resolution to implement the same. Name the type of plan discussed in the
case.
a. Strategic plans b. Tactical plans
c. Operational plans d. Single-use plans
e. Standing plans
After Chanda Kochhar joined ICICI Bank, organization started rethinking. Organization decided not to
hire former employees, not to sack employees even during recession, underperforming employees
were declared ineligible to increments, decided to continue giving medical support till 60 years of
age even after leaving the organization to boost up the morale and improve the performance of
employees. Which aspect of the organization has got changed?
a. Budgets b. Policies
c. Programs d. Procedures
Projects
7
Principles of Management
An organization planned to open a branch in foreign country within two years. It has instructed its
executives to survey market potentials in the first year and select the location and organize
resources the next year. Name the type of plan based on the time-frame.
a. Short-term plan b. Long-term plans
c. Intermediate-term plan d. Very short-term plan
e. None of the above
Exploring the craze for eco-friendly plastic vessels, Nidhi wanted to start one such manufacturing unit.
Her objective was to deliver goods as per customers’ tastes at lowest price. She has selected three
different places to establish the plant. While one location was near to the finished goods supplies,
the other location was near to raw material supply. The third location was suitable for the
availability of cheap labor. She started analyzing all the three alternatives to choose the best one.
Where exactly she is in the process of planning?
a. Analyzing opportunities b. Determining planning premises
c. Establishing objectives d. Evaluating available alternatives
Identifying alternatives
A Bank manager at a branch decided to improve its operations to foster the process and reduce the
service time. He collected suggestions of customers through suggestion box and questionnaires.
He requested his employees to suggest effective ways to improve the performance of bank during
meeting. Consequently,he came up with an innovative and effective plan that could achieve his
objective. Identify the systems approach that helped him in effective planning.
a. Management Information System b. An Open Systems Approach
c. Planning Premises d. Closed systems approach
e. None of the above
Sirish planned to open a computer training institute to earn lump sum by offering latest technologies
expected to be in the market shortly . In order to start the program, he started collecting
information on the technology and best personnel available to teach those technologies. It took
him almost six months to complete the activity. By then many institutes emerged in the area
competing with him. Name the planning limitation that shattered his expectations.
a. Lack of accurate information b. Inflexibility
c. Resistance to change d. Environmental constraints
e. Time consuming process
In a manufacturing company, the supervisor passed a circular asking employee to work two hours extra
a day to complete the production schedule by next week. Workers refused to do so as it was
against their agreement with the management and the change was without their consent. What do
you call this situation in an organization?
a. False sense of security b. Reluctance to establish goals
c. Lack of ability and commitment d. Resistance to change
Environmental constraints
8
Multiple Choice Questions
UNIT 5 – MANAGEMENT BY OBJECTIVES
Karan, a production manager of AGK group attended a meeting with CEO along with the heads of other
departments to discuss targets for the coming assessment year. She was worried about the target
assigned as the productivity level of employees was continuously falling. She then called her team
to convey the target and invite suggestions and set individual targets. They decided to follow up
every week with feedback. Finally they could meet the target. Identify the type of management
followed by her.
a. Management by objectives b. Managementby exception
c. Organization by objectives d. Organization by Exception
e. Management by Goals
Karthik is a CEO of Creative Minds Ad agency. One day he called his manager and informed about his
goals for the next year and asked him to set objectives and means to achieve those objectives
discussing with the employees. Name the hierarchy of objectives.
a. Top-down approach b. Vertical approach
c. Bottom-up approach d. Diagonal approach
e. None of the above
Organizations would first develop overall goals, next establish specific goals for various departments,
subunits and individuals. Then it would formulate action plans, implement and maintain self-
control through periodical reviews and appraises. These statements are explaining about the
__________.
a. MBO characteristics b. MBO process
c. Benefits of MBO d. Limitations of MBO
e. Concepts of MBO
In MBO program, employees are encouraged to set their own objectives and asked to show
__________________ to achieve the goals.
a. Satisfaction b. Motivation
c. Management d. Commitment
e. Clarity
The marketing VP of XYZ company observed that national sales went down by 9% for the last quarter.
When the president demanded a remedy, fearing to lose his job, he instantly increased national
advertising which improved sales for that moment. But while doing so he ignored its effect on the
long term. Identify the area focused by the VP in the case.
a. Failure to teach MBO philosophy b. Inflexibility
c. Failure to give guidelines to goal setters d. Difficulty in goal-setting
e. Emphasis on short-term goals
Mr. Bansal, manager of an online business first set organizational goals discussing with his subordinates
and employees. Later, he set goals for individuals in each department and formulated action plans.
He extended all his support required by his employees during the process. He gave themtime for
six months and reviewed the work on the last day. But to his surprise, he saw some gaps in the
performances. If followed, which step could have helped him in making his MBO process
effective?
a. Top management support b. Training for MBO
c. Formulating clear objectives d. Encouraging participation
Effective feedback
9
Principles of Management
Bhargav, working as head of a super market was given a target to increase the number of customers by
his superior under MBO process. On the day of performance review, his superior found that there
was only a marginal increase as against the set target. The superior being dissatisfied, asked
Bhargav to give an explanation and cut his incentives for that quarter. What aspect of the MBO
helped supervisor in taking the action?
a. Basis for organizational change b. Encouragement of Personal commitment
c. Development of effective controls d. Better managing
e. Clarity in organizational action
Manish, a team leader in a software company first set individual goals to all his teammembers. He then
started reviewing the performance periodically. After two weeks, he found that one of his team
members Mr. Nisheeth was not committed due to which the overall team’s performance was
lagging behind. Mr. Manish could overcome this situation by_______.
a. Encouraging participation b. Training for MBO
c. Formulating clear objectives d. Effective feedback
e. None of the above
Karthik has joined newly as an executive in a multinational company. He was asked to prepare list of
objectives along with his subordinates for the next assessment year. He was surprised as to why
his subordinates should be included in setting objectives as his earlier boss never asked himto do
so. On inquiring his senior manager, he was told that organization practices MBO. Identify the
limitation factor of MBO that confused Karthik in the new office.
a. Emphasizing on short-term goals b. Failing to teach MBO philosophy
c. Inflexibility d. Failing to teach goal setting
e. None of the above
The MBO process should receive continuous support from__________managers forits successful
implementation.
a. Top-level b. Middle-level
c. Low-level d. First-level
Last-level
10
Multiple Choice Questions
UNIT 6 – STRATEGIES, POLICIES AND PLANNING PREMISES
Mary, CEO of a confectionery planned to break into a new overseas market. She started developing
relationships with overseas suppliers, identified network of retail outlets, conducted market
research to identify consumer needs and finally found location for overseas sales team head
quarter. All these activities reflect the organizations’________.
a. Policy b. Goal
c. Guidelines d. Strategy
e. All the above
A woman shoe company set its mission statement as “we put shoes on the feet of women who are style-
conscious while insisting on health and ease”. Acc ordingly, the marketing manager suggested
advertising the company’s shoes in high-fashion health stores for the first time. How could the
marketing manager come up with such an idea?
a. Giving direction to organizing b. Giving direction to planning
c. Giving direction to staffing d. Giving directions to operationalplanning
e. None of the above
Nestle company introduced its products in Pakistan with full advertising on all media in 2012. It
involved media, advertising and campaigns for promotions. The company’s financials for the first
quarter ending March 31, 2012 showed a profit of Rs.1.66 billion. They evaluated their business
portfolios (Mineral water, Milkpack, Noodles and chocolates/juices) in two dimensions i.e.
market growth rate and relative market share. The results were displayed as follows:
Product Name SBU
Market Growth Relative
Rate Market Share
Mineral Water Star 85% 89%
Milkpack Cash cow 55% 65%
Noodles Question Mark 75% 44%
Chocolates/Juices Dog 40% 38%
Identify the corporate-level strategy followed by Nestle.
a. The value-based approach b. The strategy portfolio approach
c. The corporate portfolio approach d. All the above
e. None of the above
Reliance Industries Limited manages textile fabrics and a range of petrochemical products. For each
product group, the nature of market in terms of customers, competition, and marketing channel is
differentiated. It plans different strategies for its different product groups. Each Strategic Business
Unit (SBU) builds its own strategies to utilize its resources effectively. Name the organizational
strategy used by the Reliance Industries.
a. Corporate-level strategy b. Functional-level strategy
c. Department-level strategy d. Business-level strategy
e. All the above
Nokia came to India to exploit the diverse market, especially the unexploited market in rural India. Its
mission was to provide the customer with durable and low cost cell phones. It formulated
marketing objectives to achieve its goals by conducting SWOT analysis. After analyzing the
resources and environment to operate, its major strategy was to continue as No.1 position in
India’s rural and urban market. It adopted low cost strategy for rural India and differentiation
strategy for urban India. Name the process that Nokia adopted to capture the Indian market.
11
Principles of Management
a. Strategic planning process b. Administration process
c. Crisis management process d. Competitive process
e. None of the above
A quality inspector in a manufacturing organization proposed a strategy to adopt latest safety measures
to which the management agreed. Accordingly, he wanted to inform his workers about the latest
safety measurement tools procured by the organization. He sent a circular to the manufacturing
unit specifying the list of tools and procedures to use in writing. To that effect, he also sent some
pictures to display at the unit. Name the strategy that is used by the Quality inspector.
a. Communication strategy b. Marketing strategy
c. Sales strategy d. Project strategy
e. None of the above
A service business unit initiated a strategy to retain its existing customers and increase sales. It changed
the compensation system and the targets for its managers. It discarded the old practice of
measuring senior managers by revenue and margin targets and fixed 20 percent of their
compensation to achieve the retention goals. It also offered other bonus packages. These
initiatives taken by the company encouraged managers to make the strategy successful. Identify
the factor that helped the organization in its strategy implementation.
a. Technology b. Decision processes
c. Structure d. Maintaining strategic control
e. Reward systems
Goodwin Company manufactures and sells domestic cleaning products. Its target was to earn a profit of
billion dollars in household business in one year. But due to new entry in to the bus iness, it was
able to earn only half a billion dollars through its business in household. Margerett, the sales
manager quickly observed that there was huge potential of generating profits in retail market and
decided to shift the focus to contract packaging and distribution in industrial and automotive
cleaning markets. This helped her to meet the target of the organization. Name the strategy that
helped her to achieve the target.
a. Communication strategy b. Implementation strategy
c. Contingency Strategy d. All the above
e. None of the above
Bhargav, a General Manager was developing plans to enter overseas market that was set as a strategic
vision by the company. After a SWOT analysis, he chose a specific country but found that several
competitors with similar products have entered that market recently. Bhargav after discussing
with the Superiors decided to choose the promising opportunity of introducing a new product. He
developed plans to challenge the competitors through marketing efforts to introduce the new
product to challenge competitors successfully. Identify the element of planning that helped
Bhargav to meet the competition.
a. Organization premises b. Planning premises
c. System premises d. Market premises
Produce premises
In the above case 59, Bhargav took the help of his subordinates in exploring the overseas market and
coming up with a new challenge to introduce a new product in place of similar products. Name
the factor that helped him to effectively implement the strategy.
Developing an appropriate fit between organizational structure and planning needs
Creating a proper organizational climate
Reviewing strategies regularly
Ensuring action plans to reflect major objectives and strategies
Maintaining strategic control
12
Multiple Choice Questions
UNIT 7 – MANAGERIAL DECISION MAKING
Violet Jones is a manager at the fast food restaurant. She got a mail from the headquarters to increase
her monthly profits. She determined to select the best path to save money and decided to cut costs
in other ways than eliminating employees. She chose other alternatives such as finding cheaper
vendors, reducing store hours, changing menu options, etc. Violet evaluated all alternatives and
after much consideration, decided to submit her recommendations to management with the hope
that it will increase store profits. Name the decision-making process described in the case.
a. Rational decision-making b. Irrational decision-making
c. Garbage-can decision-making d. Satisficing decision-making
e. Incremental decision-making
Hari went to purchase a best model car available in Hyderabad that could suit his budget, give him all
comforts, have the maximum seating capacity and belongs to a top brand. At the showroom, he
found that there are no models that can meet all his demands and finally he had to choose the one
that best suited his budget. Identify the decision-making model used by Hari.
a. Rational model b. Incremental model
c. Garbage-can model d. Satisficing model
e. Programmed decision model
From several decades, customers of fast food restaurants were concerned with the taste and the price of
food rather their health. In 2005, Mc Donald encountered an unusual problem when customers
showed growing concern with regard to foods high in fat and calories. In response, McDonald’s
started offering healthy food by substituting apple slices in Happy meals to French fries and
stopped using trans-fats. Identify the decision making approach followed by McDonald in the
above case.
a. Programmed decision-making b. Non-programmed decision-making
c. Incremental decision-making d. Garbage-can decision-making
e. Bounded decision-making
To introduce a new caffeine-free cola, the vice-president of marketing for a Cola company was
considering several plans. One plan was expected to cost him 10 crores while the other to cost
only 5 crores. As per his estimations there was an equal chance of acceptance of the new cola
both the ways and profits were also marginal. He conducted expected monetary value analys is to
decide about the type of plan to be implemented. Determine the decision-making category applied
in the above case.
a. Decision-making under certainty b. Decision-making under risk
c. Decision-making under uncertainty d. Decision-making under stability
e. Decision-making using preference or utility theory
Prateek, a manager of a production floor that has different lines of production wanted to keep track of
labor costs. He started collecting and comparing the pay roll reports to the budget set by the
organization in order to implement corrective actions, if necessary. These reports were required
every week. He observed that the time taken to complete this process is eating up most of his
valuable time and wanted to adapt a system that could help him with speed, precision and
economy in carrying out this function effectively. Kindly help him with the right suggestion from
the following alternatives.
a. Management Expert System b. Transaction Support System
c. Management Decision System d. None of the above
Management Information System
13
Principles of Management
ABZ Engineering company is committed to a specific decision making approach. Whenever the
company calls for interviews, eight to ten employees are involved in interviewing every job
candidate. Also when the company has to take major decisions, it forms a committee involving
employees from all the levels of management to address the issue. Name the approach followed
by the company.
a. Group decision-making b. Expert decision-making
c. Operational decision-making d. Individual decision-making
e. Strategic decision-making
Vice President of a large manufacturing company planned to replace technology for manufacturing a
new product line. He selected members of the organization, plant managers, executives,
consumers and some renowned experts in the field to discuss on the matter. He then collected the
opinions of all the experts to calculate average and take the decision based on the results. Name
the technique used by the Vice President to take the decision in the above case.
a. Nominal group technique b. Interacting group technique
c. Delphi group technique d. Break-even analysis technique
e. Ratio analysis technique
After renovating the restaurant, a restaurant manager observed that the customer arrivals have increased
and some of the customers are kept waiting for long time to serve. He decided to increase the
number of servers. He then took the help of an operation technique to suit the number of servers
to the increased customer demand. Identify the technique used by the restaurant manager fromthe
list given below:
a. Linear programming b. Game theory
c. Simulation d. Queuing theory
e. Replacement theory
A manufacturer wanted to sell a product. He had two options before him. The product can be either
made inside the factory or purchased from outside. While producing inside would require using
some of the scarce resources, procuring from outside would not. To achieve his goal of
minimizing the total cost, he used a technique to come out with the best solution. He was
successful by adopting such technique. Name the technique that helped him in making the best
choice.
a. Delphi group technique b. Simulation theory
c. Linear programming technique d. Critical path method
e. Ratio analysis technique
Neeta was interested to invest in a company that can assure good returns. She selected some companies
to test their past performances before investing. She started calculating the long term solvency,
fixed assets to long term and debt percentages of all the firms and then compared their findings.
She then selected a company whose financial condition was stable and invested. Name the
analysis used by Neeta in this case.
a. Ratio analysis b. Marginal analysis
c. Financial analysis d. Comparative analysis
Operations analysis
14
Multiple Choice Questions
UNIT 8 – FUNDAMENTALS OF ORGANIZING
A Construction company manager is responsible for ensuring the project completion on time. For this,
he has to appoint the best subcontractors and see that building materials reach the project place
within the stipulated time and ensure proper utilization of resources and eliminate duplication of
efforts. Identify the function of management the manager is performing in the construction
company.
a. Planning b. Marketing
c. Organizing d. Controlling
e. Staffing
Karthik, a floor manager in a production department who has recently joined, examined the floor
management at the manufacturing plant. He insisted the supervisor to display the flow of work
activities being carried out at each floor. He conducted a meeting and informed the workers about
the channel of communication to approach for any clarification. These efforts of the manager have
brought improvement in the productivity of the workers. Identify the aspect of organizing that is
explained in the case.
a. Benefits of organizing b. Limitations of organizing
c. Traditional perspectives on organizing d. Formal organization
e. Informal organization
A furniture manufacturer got an order for 300 lecture stands. He has five workers to do the job. There
are two ways to complete this order. One is every worker can be asked to manufacture 60 lecture
stands. The other method is to distribute different parts of the lecture stands, like top board, center
support, assembling and polishing individually to different workers. The manufacturer strongly
believes in a management principle and has adopted the second method. Name the principle the
manufacturer believed in.
a. Chain of command b. Span of control
c. Hierarchy of authority d. Delegation of authority
e. Division of labor
An electronics supplies owner who observed downtrend in his sales volumes, immediately planned to
collect feedback from his customers on the parameters of quality and the expected changes in the
products. He then started supplying the information to the manufacturers to update their products
as per the customer’s requirements. This approach helped not only the shop owner to improve his
sales but also enhanced the quality of manufacturers through innovations. Name the approach that
helped both the owner and the organization to increase their sales.
a. Closed system b. Open system
c. Stable system d. Unstable system
e. None of the above
Get-well, a super specialty hospital is located in an industrial Area. It has 160 beds capacity. It has 40
duty medical officers and 300 Para medical staff. It recently started 24 hours medical, surgical
and cardiac emergency services. The team of medical officers, consultants and para medical staff
serves the patients throughout the day by segregating themselves into three shifts. It makes its
services available to customers round-the-clock. What form of departmentation is the hospital
following?
a. Departmentation by numbers b. Departmentation by simple numbers
c. Departmentation by time d. Departmentationby process
Departmentation by equipment
15
Principles of Management
A soap manufacturing company on being successful in achieving its target of highest market sh are in
national markets started manufacturing cleaning products for domestic and industrial purpose.
The General Manager-operations now is facing difficulty in managing the three lines of products
due to increase in the volume of operations. Now the organization is planning to reduce the
burden of its operations manager through departmentation. Identify the best suitable
deparmentation for the organization.
a. Departmentation by products b. Departmentationby process
c. Departmentation by customers d. Departmentation by territories
e. Departmentation by functions
A Corporate bank whose services are spread across territorial regions has now planned to further
provide its services by offering separate departments for commercial loans, installment loans,
savings accounts, and checking accounts. The bank has planned to departmentalize its services
based on ______________.
a. Functions b. Territory
c. Process d. Products
e. Equipment
3M, an innovative company is a leading supplier for electronic products and protective products. At 3M,
everyone is welcome to participate in innovation irrespective of their levels in the management. It
offers best challenges to innovators who can convince themselves first and able to convince
several levels of management about the value of those ideas. Name the approach followed by 3M
in innovation.
a. Top-down approach b. Bottom-up approach
c. Diagonal approach d. Cross sectionalapproach
e. Virtual approach
Departmentation by ____________________ is a type of departmentalization which is widely used in
manufacturing organizations where people and material are brought together to carry out a
particular operation. It provides the benefit of specialization and facilitates the optimum use of
resources.
a. Process or equipment b. Product
c. Territory d. Function
e. Time
Sphoorthy and Sneha, working for a company, became friends through regular meetings at lunch time in
the canteen. Both of them are working under different teams of the same project. When Sphoorthy
was in trouble to complete her task, Sneha helped spontaneously with her expertise and
experience. What type of organization has helped Sphoorthy in overcoming her problem in the
company?
a. Formal organization b. Stable organization
c. Flexible organization d. Informal organization
Closed organization
16
Multiple Choice Questions
UNIT 9 – STRATEGIC ORGANIZATION DESIGN
Ritesh is a sales executive at one of the branches of Machine Tools Supply Company. One day, his sales
representatives reported a customer complaint regarding the quality of the tools. Ritesh forwarded
the complaint to his manager and manager in turn passed it to the central manager. Ritesh has
been following up since then. But It was almost a fortnight from the day the complaint was
registered and there was no solution offered and the customer returned the tools being dissatisfied
by the delay. What aspect of management was responsible for the delay in the case discussed
above?
a. Tall structure b. Flat structure
c. Broad structure d. Wide structure
e. Large structure
Muthuraman, started a Steel Manufacturing company and was successful in gaining market share in his
state. With the success, he started branches in other states and retained the power of decision -
making with him. Slowly branches started improving sales and that demanded most of his time
for decision making. To reduce the overburden, he gave some guidelines on decision-making to
the branch manager and asked them to take charge. Name the factor of effective s pan of
management Muthuraman has planned in the case.
a. Use of objective standards b. Rate of change
c. Clarity of delegation of authority d. Clarity of plans
e. Trained subordinates
Kannan has started a BPO with a staff of 50 numbers. He used to take care of sales, marketing,
accounting, HR and IT manager responsibilities all alone. When the company started progressing,
number of staff got increased to 300 and he was unable to manage the financial aspects of the
company. He then decided to appoint an accountant to take care of company accounts. Which
factor among the effective span of management helped Kannan to reduce his burden.
a. Organizational levels b. Amount ofpersonalcontactneeded
c. Use of staff assistance d. Communication techniques
e. Trained subordinates
Srinivas is working as VP for an Automobile Sales company. He sets targets to his sales managers of
different branches. In terms of growing need of demand, he informed them that those who meet
the sales target would be given incentives as a percentage over the generated profits and those
who do not meet would be demoted. Name the power that the VP used in granting incentives to
successful managers.
a. Expert power b. Legitimate power
c. Referent power d. Reward power
e. Coercive power
In case no.84, name the power used by the VP to demote the unsuccessfulmanagers.
a. Legitimate power b. Reward power
c. Coercive power d. Expert power
e. Referent power
Caterpillar Inc., an American corporation designs, manufactures and sells machinery and engines. At
Caterpillar Inc., price related decisions were made only at the corporate headquarters. When a
sales representative working in Africa wanted to give a discount on a product, he had to check
with headquarters. Headquarters took more than a week’s time to check the markets for
information to take effective decision. The delay in decision making made them to lose their
customers. What type of management was Caterpillar following?
17
Principles of Management
a. Centralized b. Decentralized
c. Flat d. Tall
e. Informal
Home Store is a retail giant with 2,000 stores across India. At each store, managers were given
responsibility to manage a team of sales and meet the sales target. They were also responsible to
handle complaints and resolve the problems of customers. To facilitate quick decision making,
managers were also given authority to act autonomously. Name the type of authority the
management is following?
a. Centralized b. Functional
c. Formal d. Informal
e. Decentralized
Preethi, a HR manager in a multinational company was deeply engaged in a thought on how to solve the
problem of an employee. Her personal assistant observed the intensity of the problem and
recommended some alternative solutions to her. Though the solutions were feasible and
acceptable, Preethi refused her recommendations and shouted at her assistant not to involve in her
job. Name the factor of line and staff conflict that affected Preethi.
a. Viewpoint of line managers b. Lack of accountability
c. Encroachment of line authority d. Dilution of authority
e. Theoretical bias
Charan who joined as an incharge for billing counters of big super market was promoted as supervisor
looking at his relationship maintenance and control competencies. Cons equently, the branch
manager was asked by the management to explain him the job profile of supervisor and help him
in executing his tasks. Subsequently, whenever Charan approached the manager, he simply
refused to help him fearing that Charan may excel in his job and become a good contender for
him. What factor of delegation is seen affected in the case.
a. Love for authority b. Fear ofsubordinates’advancement
c. Fear of exposure d. Attitude towards subordinates
e. Personality traits and experience of the superior
Continuing the case no.89, for several months Charan managed to work without proper guidance from
the manager. Finally, unable to sustain, he submitted his resignation to the management. Identify
the factor that forced Charan to resign.
a. Lack of information and resources b. Fear of criticism
c. Lack of self-confidence d. Absence of rewards and incentives
Fear of exposure
18
Multiple Choice Questions
UNIT 10 – STRATEGIC ORGANIZATION STRUCTURE
Sheela hired a consulting firm to conduct a market comparison of staff salaries before she decided about
promotions. Soon after, a rumor started circulating through the organization that a specific group
of professionals were going to have their salaries cut as a result of comparison. Staff members did
not ask their managers. The rumor spread like wild fire. Immediately, Sheela called an emergency
staff meeting and addressed the issue by providing with the facts and gave an opportunity to the
employees to speak. What do you call this kind of communication in an organization?
a. Formal communication b. The Grapevine
c. Upward communication d. Downward communication
e. Cross communication
When DuPont manufactured gunpowder, individuals involved in the process got specialized. The
process resulted in developing separate departments for manufacturing, sales and R&D. The
functional heads were reporting directly to the Company President or CEO. To coordinate, sales
manager in New York had to interact with other sales managers than with the manager of New
York-based plants. Coordination between the plant manager and the sales manager had to occur
through the corporate head office. Name the structure followed by DuPont in the case.
a. Functional structure b. Divisional structure
c. Matrix structure d. Hybrid structure
e. SBUs
In case no. 92, which of the following factors benefit the individuals?
a. Coordination b. Monotony
c. Specialization d. a & c
e. a & b
In continuation to case no.92, once DuPont entered the diversified range of products, its coordination
requirements got changed. The production manager of DuPont’s main Dulux paint plant started
coordinating with the sales manager for paints and the other functional heads responsible for
paints only, than with any other functional head. To facilitate coordination, DuPont changed its
structure to ____.
a. Geographical structure b. Matrix structure
c. Divisional structure d. Functional structure
e. None of the above
In case no.94, while the divisional structure (product division) promoted coordination, it increases the
problem of _______at the top management level.
a. Control b. Delegate
c. Organize d. Plan
e. Review
A large conglomerate organization is composed of 90 divisions such as ConAgro, ConFood etc. The
chief executive officer faced difficulty in managing the efforts for implementing the strategies. He
then decided to divide the many divisions into individual units that has a distinct mission,
strategies and objectives and named them as i) Food Service (restaurants) 2) Retail (grocery
stores) and 3) Agricultural products. The CEO transformed its structure from divisional to
______________.
19
Principles of Management
a. Matrix structure b. Functional structure
c. Geographical structure d. Hybrid structure
e. Strategic Business Unit
In case no. 96, each individual business unit named specifically act as a distinct___________.
a. Business strategy b. Business entity
c. Business Plan d. Business decision
e. Business group
Philips Electronics was founded in 1892. Philips first focused only on the production of light bulbs.
Later it broadened its product lines across different countries. The formal corporate-level structure
was built based on the geography and product lines. With growing market changes, Philips
realized the need to restructure to cope with the changing global demands. It established national
organizations (NOs) with product divisions (PDs), and operated successfully through a network.
The network was managed by a number of coordinating committees. These committees helped in
resolving the conflicts. What do you call the organizational structure followed by Philips?
a. Matrix structure b. Customer division
c. Product division d. Geographic division
e. Hybrid structure
The coordinating committees through networks helped the individuals across the organization to work
as ________.
a. Unit b. Team
c. Panel d. An association
e. Entity
HP's operations are organized into seven business segments; They are Enterprise Storage, Servers and
Networking ("ESSN"), HP Networking, a combination of HP ProCurve and 3Com, (now part of
ESSN), HP Enterprise Services ("HPES"), HP Enterprise Security Services ("HPESS"), HP
Software Division, HP Technology Services (HPTS), Personal Systems Group ("PSG"), Imaging
and Printing Group ("IPG"), HP Financial Services ("HPFS"), and Corporate Investments. Each
business segment is organized based on the activities or functions of an organization. What do
you call the type of structure adopted by HP?
a. Functional structure b. Divisional structure
c. Matrix structure d. Hybrid structure
None of the above
20
Multiple Choice Questions
UNIT 11 – EFFECTIVE ORGANIZING AND
ORGANIZATIONAL CULTURE
Kshitij owns a chemical supplies company. Currently he has a team of 30 members working for his
company in different positions. Kshitij is planning to invest the profits he earned in further
expansion of business by starting new branches. He started estimating the future personnel needs
and the type of proposals for training, if any for expansion. These activities can make an
organization effective by_____________.
a. Listing Prerequisites b. Avoiding mistakes
c. Avoiding organizational inflexibility d. Avoiding conflict by clarification
In mobile phone industry, efficient logistics processes and manufacturing capabilities are considered as
important success factors. Nokia was indirectly underestimating its own existence
in spite of significant process. It deeply failed to consider adaptation, culture, economic
environment, creative destruction, leadership and sustainability. Nokia realized its core
competency only after forming alliance with Microsoft, when it has all the capabilities. What
factor could be the reason for Nokia’s failure in the changing market environment?
a. Organizational flexibility b. Organizational conflict
c. Organizational relationship d. Organizational culture
e. Organizational inflexibility
In case no.102, Nokia could sustain through________________.
a. Reorganization b. Reengineering
c. Reinvention d. Recollection
e. Remodification
Meenal is a General Manager for a tool manufacturing company. She instructed her sales team to
directly communicate with the development team to reduce the time delays in modifying its
designs as per customer’s requirement. As the company expanded, many functional teams
emerged and some of the employees were reporting to different managers. These changes created
chaos and confusion in reporting systemdue to absence of concerned manager’s instructions. The
organization could come out of chaos with the help of a definite _______________.
a. Organizational chart b. Functional chart
c. Organizational culture d. Functional model
e. Organizational model
Southwest Airlines CEO Herb Kelleher promotes informality and fun at workplace. His practice of
acknowledging births, marriages and deaths by sending notes and cards shows how employees are
valued. In response, the staff encourages at check-in and thus the time consumed for checking
takes less than half the industry average. These characteristics derived from its founders depict
Southwest Airline’s__________.
a. Social culture b. Organizational structure
c. Organizational culture d. Organizational rules
e. Organizational programs
Wal-Mart founder Sam Walton believed that giving respect to staff builds their self confidence. Walton
addresses his staff by their first name and always supports change to sustain in the competitive
environment. Which characteristic of organizational culture did Walton promote to maintain the
competitive edge?
a. Promoting dominant and stable values b. Being distinctive
c. Shaping philosophy and rules d. Leading to common behavioral aspects
Strengthening through visibility
21
Principles of Management
The HR manager of Corning, Inc., a manufacturing firm wanted to improve productivity. He focused on
reducing the training period and voluntary turnover of new employees. Accordingly the
orientation program was designed to foster understanding about the company’s values and to
build a positive attitude towards the company. As a result newcomer turnover was reduced by
69%. The example of Corning illustrates ____in the socialization process.
a. Step i b. Step iii
c. Step vi d. Step vii
e. Step v
In 1983, Oprah Gail Winfrey got her dream job of hosting a talk-show and soon became one of the most
famous and most professional people in the entertainment industry. In 1988, she started the
company Harpo Studios. Her television career helped the business to grow by giving her a huge
advertising. The company has more than 250 employees now and is growing every year. She also
co-founded oxygen media that helped in attracting more than 50 million viewers. Because of the
skills she possesses, she has succeeded as _________________.
a. An intrapreneur b. An entrepreneur
c. An innovator d. A manager
e. A leader
W.L Gore of Gore-Tex fabric gives his employees 10% of their work day to spend on creativity and
personal projects. One employee, Dave Myers, discovered that one of their products,
ePTFE(coating for push-pull cables) could be used to coat guitar strings. The coated strings were
more comfortable and durable than conventional guitar strings. W.L. Gore launched those strings
with a brand name ELIXIR Strings which are now the leading guitar strings sold in the market.
W.L. Gore stood as an example for ______________.
a. Intrapreneur b. Entrepreneurship
c. Executive d. CEO
e. Chief Executive
Patrick Naughton, a developer in Sun was planning to leave the company in 1995 believing that their
management was missing out the demands of PC consumer market. Then he was convinced to
continue and develop a group dedicated to the consumer market. Later, this group where James
Gosling was a member created a program called object-oriented programming language. It was
initially called Oak and later renamed as Java. The above case illustrates that it is the manager’s
responsibility to build ___________ that facilitates the achievement of group goals.
a. Energy b. Enthusiasm
c. Environment d. Ethics
Empowerment
22
Multiple Choice Questions
UNIT 12 – HUMAN RESOURCE MANAGEMENT AND STAFFING
Rahul is working as HR manager in a Steel company where the number of current workers is 300. He
received a mail from head office to plan for the expansion of production capacity by recruiting
skilled workers and a competent supervisor to manage. Keeping the strategic plan of the company
in view, Rahul started estimating the size and composition of the future work force. He started
preparing the list of skills, competencies and experiences they need to possess. What do you call
the process where Rahul is currently engaged in?
a. Forecasting manpower supply b. Forecasting manpower demand
c. Human resource planning d. Human resource actions
e. Human resource management
Continuing the case no.111, Rahul first planned to check the current computerized database for the
availability of competent and able employee who could take up the position of supervisor. What
do you call this means of assessment in human resource planning ?
a. Skills inventory b. Succession planning
c. Replacement planning d. Human inventory
e. Management inventory
Continuing the case no.112, Rahul’s efforts to trace the competent individual from the current
employees went futile. He then decided to give an advertisement in the newspaper and electronic
media. Recruiting people through these resources is called ____________.
a. Internal labor supply b. Global labor supply
c. External labor supply d. Selection
e. Placement
After completing Engineering graduation, Meenal went to attend an interview in a software company.
She completed all the rounds of interview and at last her name got displayed in the final list. She
was asked to report the next Monday. On the day of joining, she is given a list of documents to
sign an agreement where all the conditions of organization such as working hours, lunch timing,
facilities available and the promotions list including submitting a bond for one lakh rupees in case
of violating the agreement of job for two years. In the above case, what are the factors that the
organization is taking care of at the time of staffing?
a. Personnel policies b. Climate in the organization
c. Appraisal system d. All the above
e. a & c only
Apple Inc., an American computer technology company is known for its innovation. It sells personal
computer products and a range of hardware products. In order to survive and react to emerging
market demand, it reviews business documents, interprets observations, and uses surveys to
measure what is needed by most of the employees within the organization. Based on the results, it
effectively implements programs to empower employees and improve their performance. What
exactly is Apple Inc. planning to do?
a. Staffing b. Performance appraisal
c. Compensation d. Layoffs
e. Training and development
In 2010, Google was facing problems with competitors like Facebook who were attracting employees
with stock options. To address the poaching problem, Google transferred a part of employee
bonus into the basic salary. It also granted 10% salary increase and offered $1000 bonus in the
form of cash. All these initiatives are a part of "competitive _____________."
23
Principles of Management
a. Training plans b. Compensation plans
c. Appraisal plans d. Promotion plans
e. Demotion plans
Software companies today are approaching educational institutes such as universities, university
affiliated colleges and business colleges to recruit potential employees for first level positions.
What do you call this type of recruitment?
a. External recruitment b. Internal recruitment
c. Vertical recruitment d. Horizontal recruitment
e. Recruitment by referral
An education institution was offering graduation programs in management sciences. Being successful it
planned to offer post graduation programs. The top management started listing out the number of
subjects in each program to be offered and the number of available faculty for each subject. Then
it decided to appoint additional faculty members and started describing the job profiles of the
faculty members with detailed specifications. Then it gave a notification in the newspaper and
through electronic media for the requirement of faculty members. What is this process called in an
organizational environment?
a. Selection process b. Production process
c. Operation process d. Recruitment process
e. Transaction process
After qualifying a written test for the post of assistant production manager and being successful in
personal interview, Sohail was asked to attend a third round of interview at the production plant.
The interview would be headed by the production manager to whomSohail is supposed to report
on final selection. These type of interviews are called as
a. Personal interview b. Informal interview
c. Comprehensive interview d. Exit interview
e. Stress interview
Standard Chartered Bank selects management trainees from premium B- schools and conducts training
program for about six months. During the training period, the new recruiters are given an
opportunity to visit several bank divisions and meet the bank’s business heads. It is an effort made
to enable the new recruits to get an overall view of the internal operations of bank. Further, team
building sessions are also conducted for two days. Before exactly starting to work on the job, the
new recruits are given a brief description about the job. This process of introduction of new
employee to the organization is called ________________.
a. Maximization b. Minimization
c. Centralization d. Decentralization
Socialization
24
Multiple Choice Questions
UNIT 13 – PERFORMANCE APPRAISAL AND CAREER STRATEGY
McKinsey is a leading strategy consulting firm. It has a HR policy where, managers evaluate their
employees on completion of each consulting assignment in addition to the annual performance
evaluation. As such, consultants are given around 20 mini-evaluations in one year. This is done to
match with the needs of the organization and the development needs of the employee. Identify the
objective of appraisals depicted in the case.
a. Identification of potential b. Promotion decisions
c. Compensation administration d. Validation of selection procedures
e. Performance improvement
The process followed in the case no.121 is referent to which form of appraisal?
a. Informal appraisal b. Formal appraisal
c. Standard appraisal d. Systematic appraisal
e. Official appraisal
Tata Motors incorporated quarterly appraisals in to its performance appraisal system recently. The new
system facilitates employees to select their own Key Result Areas (KRAs) for each quarter and
gives an opportunity to review their performance based on these parameters. These types of
appraisals are called _________.
a. Standard appraisal b. Formal appraisal
c. Informal appraisal d. Systematic appraisal
e. Official appraisal
In Tata Motors, major revenue is generated by sales department. As such, sales people are given
increments in cash and promotions based on their performance. Rahul, who was not performing
up to the satisfaction, was given training. Consequently, his performance improved and he was
also given cash benefits. Name the quality control technique used by Tata Motors in the case, to
motivate Rahul.
a. Standards b. Information
c. Corrective action d. Performance action
e. Management action
Frito lay products include Lays, Sun chips and Doritos. It has more than 50,000 employees working out
of which one third are route sales representatives (RSR). Their job is to negotiate and put products
on store shelves. Their performance was measured through sales figures as the job was paid on
commission basis. The management observed that profitability goals were not met becaus e of low
productivity and high RSR turnover. Soon, the management detected that more than sales figures,
sales skills had the greatest impact on the sales and introduced other performance behavior based
measures to evaluate RSR such as sales task, driving and delivery tasks and merchandising tasks.
Name the latest performance management adopted by Frito lay.
a. Graphic rating method b. Management by objectives (MBO)
c. 360 degree appraisal d. Comprehensive appraisal
e. Behaviorally Anchored Rating Scales (BARS)
In a glass manufacturing company, management followed conventional output based appraisal system.
In performance appraisal meeting, the production manager identified that most of the
knowledgeable employees were not satisfied by the system. He then proposed a change in the
appraisal system where quality of work, job knowledge, dependability, punctuality and attendance
are considered in addition to output for performance appraisals. Identify the assessment method
proposed by the production manager in the case.
25
Principles of Management
a. Graphic Rating Method b. Behaviorally Anchored Rating Method
c. Forced Ranking Method d. Critical Incident Method
The Narrative Method
Production manager in a bearing company got instructions to produce 100 tonnes within three months as
against the actual capacity of 80 tonnes. The manager put all his efforts to meet the target by
giving required resources and instruction to the workers and planned to review the progress every
month. By the end of second month, they could produce 70 tonnes. Unfortunately, message
received by the manager from the Head office stated that the order has been replaced for only 80
tons which is their normal target. The efforts of the manager went unnoticed by the management
during performance appraisal. Which factor of MBO implementation played the role in
performance management?
Strengths of appraisal against verifiable objectives
Opportunities of appraisal against verifiable objectives
Weaknesses ofappraisal against verifiable objectives
Threats of appraisal against verifiable objectives
None of the above
Pratheek, a good performer as a system administrator was recommended for a training program in the
latest java development courses. But his interest was always to learn the new Hadoop technology
yet to be popular in the market. He tried to express his view to the management about his concern
but things did not work and he had to undertake the training program. Later, under the real
project, his performance was not up to the expectations. Where lays the reason for Pratheek’s low
performance in the case?
a. Implementation of a career plan b. Monitoring progress
Analysis of persona strengths and weaknesses
Consistency testing and strategic choices
Analysis of environment threats and opportunities
Tesco follows an appraisal system called 'talent planning'. This practice promotes people to work with
commitment in the organization. Individuals are allowed to apply for promotions in the areas of
their career interest through annual appraisal scheme. Their manager helps them in acquiring the
technical skills and behaviors required for these roles by conducting training sessions. This is how
Tesco achieves its business objectives and facilitates its employees to achieve their personal and
career objectives. Name the activity taken up by the Tesco managers under career strategy
formulation.
Preparation of a personalprofile
Analysis of environment threats and opportunities
Development of strategic career alternatives
d. Monitoring progress e. Implementation of a career plan
Team leader in a software company was planning to conduct a training session for his team members.
He started comparing the current competencies of his team members against the organizational
requirements and market trends to find the performance gaps. For the purpose, he also checked
their completed projects for the identification of potential training needs. But all these efforts
were time consuming, he then decided to list out such gaps through performance appraisals from
the next time. In which part of the career strategy do you realize this concept?
a. Monitoring progress b. Monitoring training needs
Development of short-range career objectives and action plans
Development of strategic career alternatives
Development of long-range personaland professionalgoals
26
Multiple Choice Questions
UNIT 14 – ORGANIZATIONAL CHANGE AND ORGANIZATION
DEVELOPMENT
In 2010, IBM introduced Accessible Workplace Connection (AWC), a process of giving special
facilities to handicapped people to complete their work. This assistive technology is also called as
a “one-stop-shop”. It allows interaction between employee s and the in charges of IBM teams
providing accommodations. AWC are established to provide continuous support by giving
reasonable and effective accommodations to eliminate process confusion. Name the factor that
forced change in the workplace environment in IBM.
a. Technology b. Economic factors
c. Competition d. Nature of the workforce
e. Political factors
An organization offering business analysis services decided to start a cost reduction program. It
searched for potential chances in the company and found most of the employees using individual
printers connected to their computer in the office. When approached IT department for an
alternative solution, they proposed replacing the individual printers with networked printers and
placing them at convenient work stations. This change was estimated to save $2 million annually.
The proposal to that effect was signed by the senior management and individual printers were
slowly being replaced. Meanwhile, majority of employees complained great inconvenience and
reported to the top management due to which the program was postponed losing the significant
opportunity costs. What is the cause for losing opportunity costs?
a. Organizational resistance b. Individual resistance
c. Group resistance d. Customer resistance
e. Stakeholder resistance
An electronic product manufacturing company wanted to eliminate unnecessary processes and improve
the use of materials by employing lean manufacturing system. In the process, it had to change the
procedures for ordering and had to lay off some of its employees. In a way the change has
affected the entire system of operations in the company leading to further cost and design
assessment. The organization temporarily put off the plans for the support from all the
departments. Which source of organizational resistance is affected in the plan?
a. Limited focus b. Threat to expertise
c. Threat to established power relationships d. Threat to established resource allocation
e. Group inertia
In case no.132, the top management gave instructions on how to motivate employees to accept the
change. Accordingly, middle level managers conducted meetings department wise and explained
the benefits of using printers through networks and the concept of cost saving. They also
explained how these benefits shall reap fruits for employees. Identify the factor that helped the
managers to overcome resistance.
a. Supervision and control b. Facilitation and support
c. Negotiation and agreement d. Manipulation and co-optation
e. Education and communication
In case no.133, Top management called for a meeting with all the heads of departments and requested to
participate in the discussion on the change consequences. The top management tried to clear the
fears about the change and asked them to come up with plans on how to implement the change
gradually with the consent of employees. Name the factor that helped the management to
overcome the resistance of change.
27
ICFAI Principles of Management- Solved assignments and case study help
ICFAI Principles of Management- Solved assignments and case study help
ICFAI Principles of Management- Solved assignments and case study help
ICFAI Principles of Management- Solved assignments and case study help
ICFAI Principles of Management- Solved assignments and case study help
ICFAI Principles of Management- Solved assignments and case study help
ICFAI Principles of Management- Solved assignments and case study help
ICFAI Principles of Management- Solved assignments and case study help
ICFAI Principles of Management- Solved assignments and case study help
ICFAI Principles of Management- Solved assignments and case study help
ICFAI Principles of Management- Solved assignments and case study help
ICFAI Principles of Management- Solved assignments and case study help
ICFAI Principles of Management- Solved assignments and case study help
ICFAI Principles of Management- Solved assignments and case study help
ICFAI Principles of Management- Solved assignments and case study help
ICFAI Principles of Management- Solved assignments and case study help
ICFAI Principles of Management- Solved assignments and case study help
ICFAI Principles of Management- Solved assignments and case study help
ICFAI Principles of Management- Solved assignments and case study help
ICFAI Principles of Management- Solved assignments and case study help
ICFAI Principles of Management- Solved assignments and case study help
ICFAI Principles of Management- Solved assignments and case study help
ICFAI Principles of Management- Solved assignments and case study help
ICFAI Principles of Management- Solved assignments and case study help
ICFAI Principles of Management- Solved assignments and case study help
ICFAI Principles of Management- Solved assignments and case study help
ICFAI Principles of Management- Solved assignments and case study help
ICFAI Principles of Management- Solved assignments and case study help
ICFAI Principles of Management- Solved assignments and case study help
ICFAI Principles of Management- Solved assignments and case study help
ICFAI Principles of Management- Solved assignments and case study help
ICFAI Principles of Management- Solved assignments and case study help
ICFAI Principles of Management- Solved assignments and case study help
ICFAI Principles of Management- Solved assignments and case study help
ICFAI Principles of Management- Solved assignments and case study help
ICFAI Principles of Management- Solved assignments and case study help
ICFAI Principles of Management- Solved assignments and case study help
ICFAI Principles of Management- Solved assignments and case study help
ICFAI Principles of Management- Solved assignments and case study help
ICFAI Principles of Management- Solved assignments and case study help

Weitere ähnliche Inhalte

Was ist angesagt?

Unit5 learning and development in knowledge setting
Unit5 learning and development in knowledge settingUnit5 learning and development in knowledge setting
Unit5 learning and development in knowledge settingRee Tu
 
Learning: Theories and Program Design
Learning: Theories and Program DesignLearning: Theories and Program Design
Learning: Theories and Program Designaizellbernal
 
Factors that affect the on the-job training of
Factors that affect the on the-job training ofFactors that affect the on the-job training of
Factors that affect the on the-job training ofSunjay Taladtad
 
Role of Learning Curve - by: Rahat Kazmi
Role of Learning Curve - by: Rahat KazmiRole of Learning Curve - by: Rahat Kazmi
Role of Learning Curve - by: Rahat KazmiRahat Kazmi
 
ED607 Assignment #1 - Assure
ED607 Assignment #1 - Assure ED607 Assignment #1 - Assure
ED607 Assignment #1 - Assure lilkitcat13
 
Educational Leadership
Educational LeadershipEducational Leadership
Educational LeadershipDr. N. Asokan
 
Handouts for Expanded Roles for Business and Community Partners
Handouts for Expanded Roles for Business  and Community PartnersHandouts for Expanded Roles for Business  and Community Partners
Handouts for Expanded Roles for Business and Community PartnersNAFCareerAcads
 
DEVELOPMENT AND APPLICATION OF LEARN AND WORK ASSIGNMENTS (LWAs)
DEVELOPMENT AND APPLICATION OF LEARN AND WORK ASSIGNMENTS (LWAs)DEVELOPMENT AND APPLICATION OF LEARN AND WORK ASSIGNMENTS (LWAs)
DEVELOPMENT AND APPLICATION OF LEARN AND WORK ASSIGNMENTS (LWAs)Ghazally Spahat
 
An examination of collegiate internships in entrepreneurial leadership settin...
An examination of collegiate internships in entrepreneurial leadership settin...An examination of collegiate internships in entrepreneurial leadership settin...
An examination of collegiate internships in entrepreneurial leadership settin...Berea College
 
Hacking Training and Development - Why Your Employee Development Program Sucks
Hacking Training and Development - Why Your Employee Development Program SucksHacking Training and Development - Why Your Employee Development Program Sucks
Hacking Training and Development - Why Your Employee Development Program SucksPatrick Bosworth
 
Training & Development - Education - Training - Development, T&D Competencies
Training & Development - Education - Training - Development, T&D CompetenciesTraining & Development - Education - Training - Development, T&D Competencies
Training & Development - Education - Training - Development, T&D CompetenciesM R Jhalawad
 

Was ist angesagt? (20)

HRM module 4
HRM module 4HRM module 4
HRM module 4
 
Unit5 learning and development in knowledge setting
Unit5 learning and development in knowledge settingUnit5 learning and development in knowledge setting
Unit5 learning and development in knowledge setting
 
Hrd 16
Hrd 16Hrd 16
Hrd 16
 
Learning: Theories and Program Design
Learning: Theories and Program DesignLearning: Theories and Program Design
Learning: Theories and Program Design
 
Factors that affect the on the-job training of
Factors that affect the on the-job training ofFactors that affect the on the-job training of
Factors that affect the on the-job training of
 
Role of Learning Curve - by: Rahat Kazmi
Role of Learning Curve - by: Rahat KazmiRole of Learning Curve - by: Rahat Kazmi
Role of Learning Curve - by: Rahat Kazmi
 
A
AA
A
 
Hrd 8
Hrd 8Hrd 8
Hrd 8
 
04 learning - theories and program design
04   learning - theories and program design04   learning - theories and program design
04 learning - theories and program design
 
ED607 Assignment #1 - Assure
ED607 Assignment #1 - Assure ED607 Assignment #1 - Assure
ED607 Assignment #1 - Assure
 
Hrd 10
Hrd 10Hrd 10
Hrd 10
 
Educational Leadership
Educational LeadershipEducational Leadership
Educational Leadership
 
Training and development
Training and developmentTraining and development
Training and development
 
Handouts for Expanded Roles for Business and Community Partners
Handouts for Expanded Roles for Business  and Community PartnersHandouts for Expanded Roles for Business  and Community Partners
Handouts for Expanded Roles for Business and Community Partners
 
Hrd 17
Hrd 17Hrd 17
Hrd 17
 
DEVELOPMENT AND APPLICATION OF LEARN AND WORK ASSIGNMENTS (LWAs)
DEVELOPMENT AND APPLICATION OF LEARN AND WORK ASSIGNMENTS (LWAs)DEVELOPMENT AND APPLICATION OF LEARN AND WORK ASSIGNMENTS (LWAs)
DEVELOPMENT AND APPLICATION OF LEARN AND WORK ASSIGNMENTS (LWAs)
 
An examination of collegiate internships in entrepreneurial leadership settin...
An examination of collegiate internships in entrepreneurial leadership settin...An examination of collegiate internships in entrepreneurial leadership settin...
An examination of collegiate internships in entrepreneurial leadership settin...
 
Hacking Training and Development - Why Your Employee Development Program Sucks
Hacking Training and Development - Why Your Employee Development Program SucksHacking Training and Development - Why Your Employee Development Program Sucks
Hacking Training and Development - Why Your Employee Development Program Sucks
 
Training & Development - Education - Training - Development, T&D Competencies
Training & Development - Education - Training - Development, T&D CompetenciesTraining & Development - Education - Training - Development, T&D Competencies
Training & Development - Education - Training - Development, T&D Competencies
 
Hrd 12
Hrd 12Hrd 12
Hrd 12
 

Ähnlich wie ICFAI Principles of Management- Solved assignments and case study help

ICFAI Organizational Behavior - Solved assignments and case study help
ICFAI Organizational Behavior - Solved assignments and case study helpICFAI Organizational Behavior - Solved assignments and case study help
ICFAI Organizational Behavior - Solved assignments and case study helpsmumbahelp
 
ICFAI - Business Environment and Law - work book
ICFAI - Business Environment and Law - work bookICFAI - Business Environment and Law - work book
ICFAI - Business Environment and Law - work booksmumbahelp
 
ICFAI Marketing Management - Solved assignments and case study help
ICFAI Marketing Management - Solved assignments and case study helpICFAI Marketing Management - Solved assignments and case study help
ICFAI Marketing Management - Solved assignments and case study helpsmumbahelp
 
ICFAI - Business Communications and Soft Skills - work book
 ICFAI - Business Communications and Soft Skills - work book ICFAI - Business Communications and Soft Skills - work book
ICFAI - Business Communications and Soft Skills - work booksmumbahelp
 
ICFAI Human Resource Management - Solved assignments and case study help
ICFAI Human Resource Management - Solved assignments and case study helpICFAI Human Resource Management - Solved assignments and case study help
ICFAI Human Resource Management - Solved assignments and case study helpsmumbahelp
 
ICFAI Organizational Behavior - Solved assignments and case study help
ICFAI Organizational Behavior - Solved assignments and case study helpICFAI Organizational Behavior - Solved assignments and case study help
ICFAI Organizational Behavior - Solved assignments and case study helpsmumbahelp
 
ICFAI Economics for Managers - work book
ICFAI Economics for Managers  - work bookICFAI Economics for Managers  - work book
ICFAI Economics for Managers - work booksmumbahelp
 
Emotional intelligence best practice
Emotional intelligence best practiceEmotional intelligence best practice
Emotional intelligence best practiceRuwan Kannangara
 
Evaluate How To Involve The Learner In The Assessment Process
Evaluate How To Involve The Learner In The Assessment ProcessEvaluate How To Involve The Learner In The Assessment Process
Evaluate How To Involve The Learner In The Assessment ProcessSusan Tullis
 
Performance Development best practices
Performance Development  best practicesPerformance Development  best practices
Performance Development best practicesLakesia Wright
 
Workshop 05
Workshop 05Workshop 05
Workshop 05republic
 
Successful strategies for social studies teaching and learning
Successful strategies for social studies teaching and learningSuccessful strategies for social studies teaching and learning
Successful strategies for social studies teaching and learningKarylle Honeybee Ako
 
presentation1inbuildingbri-130315185847-phpapp01.pdf
presentation1inbuildingbri-130315185847-phpapp01.pdfpresentation1inbuildingbri-130315185847-phpapp01.pdf
presentation1inbuildingbri-130315185847-phpapp01.pdfJenniferMayano3
 
KYT with trn + proposal
KYT with trn + proposalKYT with trn + proposal
KYT with trn + proposaljohn varkey
 
UHI Millennium Institute, HoTLS, Experiential Education Presentation, 2008
UHI Millennium Institute, HoTLS, Experiential Education Presentation, 2008UHI Millennium Institute, HoTLS, Experiential Education Presentation, 2008
UHI Millennium Institute, HoTLS, Experiential Education Presentation, 2008Rob Macpherson
 
High engagement in learning
High engagement in learningHigh engagement in learning
High engagement in learningMarek Hyla
 
ICFAI IT and Systems - Solved assignments and case study help
ICFAI IT and Systems  - Solved assignments and case study helpICFAI IT and Systems  - Solved assignments and case study help
ICFAI IT and Systems - Solved assignments and case study helpsmumbahelp
 
Self-directed learners need learning ecosystems, not courses
Self-directed learners need learning ecosystems, not coursesSelf-directed learners need learning ecosystems, not courses
Self-directed learners need learning ecosystems, not coursesSprout Labs
 

Ähnlich wie ICFAI Principles of Management- Solved assignments and case study help (20)

ICFAI Organizational Behavior - Solved assignments and case study help
ICFAI Organizational Behavior - Solved assignments and case study helpICFAI Organizational Behavior - Solved assignments and case study help
ICFAI Organizational Behavior - Solved assignments and case study help
 
ICFAI - Business Environment and Law - work book
ICFAI - Business Environment and Law - work bookICFAI - Business Environment and Law - work book
ICFAI - Business Environment and Law - work book
 
ICFAI Marketing Management - Solved assignments and case study help
ICFAI Marketing Management - Solved assignments and case study helpICFAI Marketing Management - Solved assignments and case study help
ICFAI Marketing Management - Solved assignments and case study help
 
ICFAI - Business Communications and Soft Skills - work book
 ICFAI - Business Communications and Soft Skills - work book ICFAI - Business Communications and Soft Skills - work book
ICFAI - Business Communications and Soft Skills - work book
 
ICFAI Human Resource Management - Solved assignments and case study help
ICFAI Human Resource Management - Solved assignments and case study helpICFAI Human Resource Management - Solved assignments and case study help
ICFAI Human Resource Management - Solved assignments and case study help
 
ICFAI Organizational Behavior - Solved assignments and case study help
ICFAI Organizational Behavior - Solved assignments and case study helpICFAI Organizational Behavior - Solved assignments and case study help
ICFAI Organizational Behavior - Solved assignments and case study help
 
ICFAI Economics for Managers - work book
ICFAI Economics for Managers  - work bookICFAI Economics for Managers  - work book
ICFAI Economics for Managers - work book
 
Emotional intelligence best practice
Emotional intelligence best practiceEmotional intelligence best practice
Emotional intelligence best practice
 
Evaluate How To Involve The Learner In The Assessment Process
Evaluate How To Involve The Learner In The Assessment ProcessEvaluate How To Involve The Learner In The Assessment Process
Evaluate How To Involve The Learner In The Assessment Process
 
Performance Development best practices
Performance Development  best practicesPerformance Development  best practices
Performance Development best practices
 
Portfolio Study NATCON 2008
Portfolio Study NATCON 2008Portfolio Study NATCON 2008
Portfolio Study NATCON 2008
 
Workshop 05
Workshop 05Workshop 05
Workshop 05
 
Successful strategies for social studies teaching and learning
Successful strategies for social studies teaching and learningSuccessful strategies for social studies teaching and learning
Successful strategies for social studies teaching and learning
 
232269
232269232269
232269
 
presentation1inbuildingbri-130315185847-phpapp01.pdf
presentation1inbuildingbri-130315185847-phpapp01.pdfpresentation1inbuildingbri-130315185847-phpapp01.pdf
presentation1inbuildingbri-130315185847-phpapp01.pdf
 
KYT with trn + proposal
KYT with trn + proposalKYT with trn + proposal
KYT with trn + proposal
 
UHI Millennium Institute, HoTLS, Experiential Education Presentation, 2008
UHI Millennium Institute, HoTLS, Experiential Education Presentation, 2008UHI Millennium Institute, HoTLS, Experiential Education Presentation, 2008
UHI Millennium Institute, HoTLS, Experiential Education Presentation, 2008
 
High engagement in learning
High engagement in learningHigh engagement in learning
High engagement in learning
 
ICFAI IT and Systems - Solved assignments and case study help
ICFAI IT and Systems  - Solved assignments and case study helpICFAI IT and Systems  - Solved assignments and case study help
ICFAI IT and Systems - Solved assignments and case study help
 
Self-directed learners need learning ecosystems, not courses
Self-directed learners need learning ecosystems, not coursesSelf-directed learners need learning ecosystems, not courses
Self-directed learners need learning ecosystems, not courses
 

Kürzlich hochgeladen

Inclusivity Essentials_ Creating Accessible Websites for Nonprofits .pdf
Inclusivity Essentials_ Creating Accessible Websites for Nonprofits .pdfInclusivity Essentials_ Creating Accessible Websites for Nonprofits .pdf
Inclusivity Essentials_ Creating Accessible Websites for Nonprofits .pdfTechSoup
 
Earth Day Presentation wow hello nice great
Earth Day Presentation wow hello nice greatEarth Day Presentation wow hello nice great
Earth Day Presentation wow hello nice greatYousafMalik24
 
Keynote by Prof. Wurzer at Nordex about IP-design
Keynote by Prof. Wurzer at Nordex about IP-designKeynote by Prof. Wurzer at Nordex about IP-design
Keynote by Prof. Wurzer at Nordex about IP-designMIPLM
 
Field Attribute Index Feature in Odoo 17
Field Attribute Index Feature in Odoo 17Field Attribute Index Feature in Odoo 17
Field Attribute Index Feature in Odoo 17Celine George
 
Integumentary System SMP B. Pharm Sem I.ppt
Integumentary System SMP B. Pharm Sem I.pptIntegumentary System SMP B. Pharm Sem I.ppt
Integumentary System SMP B. Pharm Sem I.pptshraddhaparab530
 
Student Profile Sample - We help schools to connect the data they have, with ...
Student Profile Sample - We help schools to connect the data they have, with ...Student Profile Sample - We help schools to connect the data they have, with ...
Student Profile Sample - We help schools to connect the data they have, with ...Seán Kennedy
 
Activity 2-unit 2-update 2024. English translation
Activity 2-unit 2-update 2024. English translationActivity 2-unit 2-update 2024. English translation
Activity 2-unit 2-update 2024. English translationRosabel UA
 
Music 9 - 4th quarter - Vocal Music of the Romantic Period.pptx
Music 9 - 4th quarter - Vocal Music of the Romantic Period.pptxMusic 9 - 4th quarter - Vocal Music of the Romantic Period.pptx
Music 9 - 4th quarter - Vocal Music of the Romantic Period.pptxleah joy valeriano
 
Choosing the Right CBSE School A Comprehensive Guide for Parents
Choosing the Right CBSE School A Comprehensive Guide for ParentsChoosing the Right CBSE School A Comprehensive Guide for Parents
Choosing the Right CBSE School A Comprehensive Guide for Parentsnavabharathschool99
 
Active Learning Strategies (in short ALS).pdf
Active Learning Strategies (in short ALS).pdfActive Learning Strategies (in short ALS).pdf
Active Learning Strategies (in short ALS).pdfPatidar M
 
Concurrency Control in Database Management system
Concurrency Control in Database Management systemConcurrency Control in Database Management system
Concurrency Control in Database Management systemChristalin Nelson
 
4.16.24 Poverty and Precarity--Desmond.pptx
4.16.24 Poverty and Precarity--Desmond.pptx4.16.24 Poverty and Precarity--Desmond.pptx
4.16.24 Poverty and Precarity--Desmond.pptxmary850239
 
Visit to a blind student's school🧑‍🦯🧑‍🦯(community medicine)
Visit to a blind student's school🧑‍🦯🧑‍🦯(community medicine)Visit to a blind student's school🧑‍🦯🧑‍🦯(community medicine)
Visit to a blind student's school🧑‍🦯🧑‍🦯(community medicine)lakshayb543
 
Difference Between Search & Browse Methods in Odoo 17
Difference Between Search & Browse Methods in Odoo 17Difference Between Search & Browse Methods in Odoo 17
Difference Between Search & Browse Methods in Odoo 17Celine George
 
Q4-PPT-Music9_Lesson-1-Romantic-Opera.pptx
Q4-PPT-Music9_Lesson-1-Romantic-Opera.pptxQ4-PPT-Music9_Lesson-1-Romantic-Opera.pptx
Q4-PPT-Music9_Lesson-1-Romantic-Opera.pptxlancelewisportillo
 
ECONOMIC CONTEXT - PAPER 1 Q3: NEWSPAPERS.pptx
ECONOMIC CONTEXT - PAPER 1 Q3: NEWSPAPERS.pptxECONOMIC CONTEXT - PAPER 1 Q3: NEWSPAPERS.pptx
ECONOMIC CONTEXT - PAPER 1 Q3: NEWSPAPERS.pptxiammrhaywood
 
ENG 5 Q4 WEEk 1 DAY 1 Restate sentences heard in one’s own words. Use appropr...
ENG 5 Q4 WEEk 1 DAY 1 Restate sentences heard in one’s own words. Use appropr...ENG 5 Q4 WEEk 1 DAY 1 Restate sentences heard in one’s own words. Use appropr...
ENG 5 Q4 WEEk 1 DAY 1 Restate sentences heard in one’s own words. Use appropr...JojoEDelaCruz
 
ENGLISH 7_Q4_LESSON 2_ Employing a Variety of Strategies for Effective Interp...
ENGLISH 7_Q4_LESSON 2_ Employing a Variety of Strategies for Effective Interp...ENGLISH 7_Q4_LESSON 2_ Employing a Variety of Strategies for Effective Interp...
ENGLISH 7_Q4_LESSON 2_ Employing a Variety of Strategies for Effective Interp...JhezDiaz1
 

Kürzlich hochgeladen (20)

Inclusivity Essentials_ Creating Accessible Websites for Nonprofits .pdf
Inclusivity Essentials_ Creating Accessible Websites for Nonprofits .pdfInclusivity Essentials_ Creating Accessible Websites for Nonprofits .pdf
Inclusivity Essentials_ Creating Accessible Websites for Nonprofits .pdf
 
Earth Day Presentation wow hello nice great
Earth Day Presentation wow hello nice greatEarth Day Presentation wow hello nice great
Earth Day Presentation wow hello nice great
 
Keynote by Prof. Wurzer at Nordex about IP-design
Keynote by Prof. Wurzer at Nordex about IP-designKeynote by Prof. Wurzer at Nordex about IP-design
Keynote by Prof. Wurzer at Nordex about IP-design
 
Field Attribute Index Feature in Odoo 17
Field Attribute Index Feature in Odoo 17Field Attribute Index Feature in Odoo 17
Field Attribute Index Feature in Odoo 17
 
Integumentary System SMP B. Pharm Sem I.ppt
Integumentary System SMP B. Pharm Sem I.pptIntegumentary System SMP B. Pharm Sem I.ppt
Integumentary System SMP B. Pharm Sem I.ppt
 
Student Profile Sample - We help schools to connect the data they have, with ...
Student Profile Sample - We help schools to connect the data they have, with ...Student Profile Sample - We help schools to connect the data they have, with ...
Student Profile Sample - We help schools to connect the data they have, with ...
 
Activity 2-unit 2-update 2024. English translation
Activity 2-unit 2-update 2024. English translationActivity 2-unit 2-update 2024. English translation
Activity 2-unit 2-update 2024. English translation
 
Music 9 - 4th quarter - Vocal Music of the Romantic Period.pptx
Music 9 - 4th quarter - Vocal Music of the Romantic Period.pptxMusic 9 - 4th quarter - Vocal Music of the Romantic Period.pptx
Music 9 - 4th quarter - Vocal Music of the Romantic Period.pptx
 
Choosing the Right CBSE School A Comprehensive Guide for Parents
Choosing the Right CBSE School A Comprehensive Guide for ParentsChoosing the Right CBSE School A Comprehensive Guide for Parents
Choosing the Right CBSE School A Comprehensive Guide for Parents
 
Active Learning Strategies (in short ALS).pdf
Active Learning Strategies (in short ALS).pdfActive Learning Strategies (in short ALS).pdf
Active Learning Strategies (in short ALS).pdf
 
Concurrency Control in Database Management system
Concurrency Control in Database Management systemConcurrency Control in Database Management system
Concurrency Control in Database Management system
 
4.16.24 Poverty and Precarity--Desmond.pptx
4.16.24 Poverty and Precarity--Desmond.pptx4.16.24 Poverty and Precarity--Desmond.pptx
4.16.24 Poverty and Precarity--Desmond.pptx
 
Visit to a blind student's school🧑‍🦯🧑‍🦯(community medicine)
Visit to a blind student's school🧑‍🦯🧑‍🦯(community medicine)Visit to a blind student's school🧑‍🦯🧑‍🦯(community medicine)
Visit to a blind student's school🧑‍🦯🧑‍🦯(community medicine)
 
Difference Between Search & Browse Methods in Odoo 17
Difference Between Search & Browse Methods in Odoo 17Difference Between Search & Browse Methods in Odoo 17
Difference Between Search & Browse Methods in Odoo 17
 
Raw materials used in Herbal Cosmetics.pptx
Raw materials used in Herbal Cosmetics.pptxRaw materials used in Herbal Cosmetics.pptx
Raw materials used in Herbal Cosmetics.pptx
 
Q4-PPT-Music9_Lesson-1-Romantic-Opera.pptx
Q4-PPT-Music9_Lesson-1-Romantic-Opera.pptxQ4-PPT-Music9_Lesson-1-Romantic-Opera.pptx
Q4-PPT-Music9_Lesson-1-Romantic-Opera.pptx
 
ECONOMIC CONTEXT - PAPER 1 Q3: NEWSPAPERS.pptx
ECONOMIC CONTEXT - PAPER 1 Q3: NEWSPAPERS.pptxECONOMIC CONTEXT - PAPER 1 Q3: NEWSPAPERS.pptx
ECONOMIC CONTEXT - PAPER 1 Q3: NEWSPAPERS.pptx
 
ENG 5 Q4 WEEk 1 DAY 1 Restate sentences heard in one’s own words. Use appropr...
ENG 5 Q4 WEEk 1 DAY 1 Restate sentences heard in one’s own words. Use appropr...ENG 5 Q4 WEEk 1 DAY 1 Restate sentences heard in one’s own words. Use appropr...
ENG 5 Q4 WEEk 1 DAY 1 Restate sentences heard in one’s own words. Use appropr...
 
ENGLISH 7_Q4_LESSON 2_ Employing a Variety of Strategies for Effective Interp...
ENGLISH 7_Q4_LESSON 2_ Employing a Variety of Strategies for Effective Interp...ENGLISH 7_Q4_LESSON 2_ Employing a Variety of Strategies for Effective Interp...
ENGLISH 7_Q4_LESSON 2_ Employing a Variety of Strategies for Effective Interp...
 
FINALS_OF_LEFT_ON_C'N_EL_DORADO_2024.pptx
FINALS_OF_LEFT_ON_C'N_EL_DORADO_2024.pptxFINALS_OF_LEFT_ON_C'N_EL_DORADO_2024.pptx
FINALS_OF_LEFT_ON_C'N_EL_DORADO_2024.pptx
 

ICFAI Principles of Management- Solved assignments and case study help

  • 1. Dear students, get ICFAI latest Solved assignments and case study help by professionals. Mail us at : help.mbaassignments@gmail.com Call us at : 08263069601
  • 2. AN INTRODUCTION TO DIFFERENTIATED LEARNING TOOLS Participants in flexible learning programs have limitations on the nature of the time they can spend on learning. Typically they are employed fully or partially, pursuing higher studies or have other social and familial responsibilities. Availability of time is a great constraint to these students. To aidthe participants,we have developedfour unique learningtools as below: Bullet Notes : Helps in introducing the important concepts in each unit of curriculum, equip the student during preparation of examinations and Case Studies : Illustrate the concepts through real life experiences  Workbook : Helps absorption of learning through questions based on reallife nuggets PEP Notes :Sharing notes of practices and experiences in the Industry will help the student to rightly perceive and get inspired to learn concepts at the cutting edge application level.placementinterviews Why are these needed?  Adults learn differently from B. School or college going students who spend long hours at campus.  Enhancing analytical skills through application related learning kits trigger experiential learning  Availability of time is a challenge.  Career success increasingly depends on continuous learning and success What makes it relevant?  How is it useful?   Where does this lead to? As and when you get 5 to 10 minutes you can read one of these and absorb and comprehend. Spending more time is your choice. You can use the time in travel, waiting for meetings, lunch time, small breaks or at home usefully. Through these tools, the learning bytes are right sized for ease of learning for time challenged participants. The content starts from practice and connect to precept making it easy to connect to industry and retain. They can be connectedto continuous assessment process of the academic program. Practitioners can use their real life knowledge and skill to enhance learning skills. Immediate visualization of the practical dimension of the concept will offer a rich learning experience.
  • 3. Easier to move ahead in the learning process.  Will facilitate the student to complete the program earlier than otherwise.Helpsstay motivated and connected. When is it useful? 
  • 5. © The ICFAI Foundationfor Higher Education (IFHE), Hyderabad,December, 2014.All rights reserved No part of this publication may be reproduced, stored in a retrieval system, used in a spreadsheet, or transmitted in any form or by any means – electronic,mechanical, ph otocopying or otherwise – without prior permission in writing from The ICFAI Foundation for Higher Education (IFHE), Hyderabad. Ref. No. PM-WB-IFHE – 122014 For any clarification regarding this book, the students may please write to The ICFAI Foundation for Higher Education (IFHE), Hyderabad giving the above reference number of this book specifying chapter and page number. While every possible care has been taken in type-setting and printing this book, The ICFAI Foundation for Higher Education (IFHE), Hyderabad welcomes suggestions from students for improvement in future editions. Our E-mail ID: cwfeedback@icfaiuniversity.in
  • 6. INTRODUCTION Participants in ICFAI University Programs are eager to learn more from practice. They realize that application orientation can enhance their learning and subsequent usage of management precepts and practices. Picking out the principle behind real world events is critical to this learning, as also identifying the alternative/solution using the principle. Towards this end the institution has reengineered the Workbook. The Workbook is a set of questions which typically illustrate a real life context from contemporary corporate happenings and then poses a question to the student for reflection. The narration of question helps the reader to reinforce the concept and facilitates the student to enhance his/her capabilities in analyzing and interpreting the conceptual frameworks. The examples depicting the names of existing persons or companies are taken from news clippings/ published articles from various public domain websites or website of respective companies. Since live examples reinforce the understanding of the students, the possible responses are connected to the concepts taught directly or indirectly. In many cases the alternatives provided are choices in a work situation based on alternative approaches. These questions provoke the learner to start thinking from the application side and connect to the knowledge that he will use to solve. Practitioners can connect better thereby improving the learning experience. This form of assessment improves learning while assessing whereas the conventional form is more about assessment of learning. The learning outcomes expected are: The examples are linked back to application of theoretical knowledge in the illustrated real-time situation. This facilitates the student to develop analytical approach in similar or related situations. Application based approach which enhances absorption and retention significantly. Exposure to the current incidences and situations in relation to important concepts of the subject. The Workbook format is also used for Assessment.
  • 7. DETAILED CURRICULUM UNIT 1: MANAGEMENT: AN OVERVIEW Definition of Management- The Role of Management - Functions of Managers: Planning, Organizing, Staffing, Leading, Controlling - Levels of Management: Top-level Managers, Middle-level Managers, First-level Managers; Time spent in carrying out Managerial Functions ᜀ Ā ᜀ Ā ᜀ Ā ᜀ Ā anagement Skills and Organizational Hierarchy: Technical Skills, Human Skills, Conceptual and Design Skills - Approaches to Management UNIT 2: EVOLUTION OF MANAGEMENT THOUGHT Early Approaches to Management: Robert Owen: Human Resource Management Pioneer; Charles Babbage: Investor and Management Scientist; Andrew Ure and Charles Dupin: Management Education Pioneers; Henry Robinson Towne (1844-1924) - Classical Approach: Scientific Management-Frederick Winslow Taylor, Frank and Lillian Gilbreth, Henry Laurence Gantt, Limitations of Scientific Management; Administrative Theory-Henri Fayol, Bureaucratic Management, Limitations of bureaucratic management and administrative theory - Behavioral Approach: Mary Parker Follet: Focusing on Group Influences Elton Mayo : Focusing on Human Relations-Illumination experiments, Relay assembly test room experiments, Interview Phase, Bank wiring observation room experiments, contributions and criticism of Hawthorne studies Abraham Maslow: Focusing on Human Needs Douglas McGregor: Challenging Traditional Assumptions about Employees Chris Argyris: Matching Human and Organizational Development, Model I and Model II organizations - Quantitative Approach: Management Science, Operations Management, Management Information Systems - Modern Approaches to Management: Systems Theory, Contingency Theory, Theory Z UNIT 3: SOCIAL AND ETHICALRESPONSIBILITIES OF MANAGEMENT Social Responsibilities of Management -Arguments for and against Social Responsibilities of Business: Arguments for Social Responsibilities of Business: Change in public expectations, Business is a part of society, Avoiding intervention by government, Balance of responsibility and power, Impact of internal activities of the organization on the external environment, Protecting shareholder interests, New avenues to create profits, favorable public image, Endeavor to find new solutions, Best use of resources of a business, Prevention is better than cure - Arguments against Social Responsibilities of Business: Opposes the principle of profit maximization, Excessive costs, Weakened international balance of payments, Increase in the firm’s power and influence, Lack of necessary skills among business people, Lack of accountability to society, Lack of consensus on social involvement -Social Stakeholders: Shareholders, Employees, Customers, Creditors and Suppliers, Society, Government - Measuring Social Responsiveness: Contributions, Fund-raising, Volunteerism, Recycling, Valuing diversity, direct corporate investment, Quality of work life, Attention to consumers, Pollution control, Social audits - Managerial Ethics - Types of Managerial Ethics: Moral management, Amoral management, Immoral management. Factors that influence Ethical Behavior: stages of moral development, individual characteristics, Structural variables, Organization’s culture, Issue intensity - Ethical Guidelines for Managers: Obeying the law, Tell the truth, Uphold human dignity, Adhere to the golden
  • 8. rule, Primum non-nocere, Allow room for participation, Always act when you have responsibility - Mechanism for Ethical Management: Code of ethics, Ethics committee, Ethics audits, Ethics training, Ethics hotline. UNIT 4: FUNDAMENTALS OF PLANNING Definitions of Planning - Nature of Planning -Significance of Planning: Focuses Attention on Objectives, Offsets Uncertainty and Risk, Provides Sense of Direction, Increases Organizational Effectiveness, Provides Efficiency in Operations, Ensures Better Coordination, Facilitates Control, Encourages Innovation and Creativity, Facilitates Delegation - Types of Plans: Plans based on Organizational Level: Strategic plans, Tactical plans, Operational plans, Plans based on Frequency of Use: Single-use plans, Standing plans, Plans based on their Time- Frame: Short-term plans, Intermediate-term plans, Long-term plans - Steps in the Planning Process: Analyzing Opportunities, Establishing Objectives, Determining Planning Premises, Identifying Alternatives, Evaluating Available Alternatives, Selecting the Most Appropriate Alternative, Implementing the Plan, Reviewing the Plan - Prerequisites of Effective Planning: Establishing the Right Climate for Planning, Clear and Specific Objectives, Planning Premises, Initiative at the Top Level, Participation in Planning Process, Communication of Planning Process, Integrating Long-Term and Short-Term Plans, An Open Systems Approach, Management Information System - Limitations of Planning: Lack of Accurate Information, Time consuming Process, Expensive, Inflexibility, Resistance to Change, Environmental Constraints, Lack of Ability and Commitment, False Sense of Security, Reluctance to Establish Goals UNIT 5: MANAGEMENT BYOBJECTIVES Nature of Objectives: Hierarchy of Objectives, The Process of Forming Objectives and Organizational Hierarchy, A Network of Objectives, Multiplicity of Objectives - Concepts of MBO: Early Impetus to MBO, Emphasis on Performance Appraisal, Inclusion of Long-term Planning in the MBO Process, The Systems Approach to MBO - The Process of MBO: Developing overall organizational goals, Establishing specific goals for various departments, subunits and individuals, Formulating action plans, Implementing and maintaining self-control, Periodic Review, Performance appraisal - Benefits of MBO: Better Management, Clarity in Organizational Action, Encouragement of Personal Commitment, Personnel Satisfaction, Basis for Organizational Change, Development of Effective Controls - Limitations of MBO: Failure to Teach MBO Philosophy, Failure to Give Guidelines to Goal Setters, Difficulty in Goad- setting, Emphasis on Short-term Goals, Inflexibility, Other Dangers - Making MBO Effective: Top Management Support, Training for MBO, Formulating Clear Objectives, Effective Feedback, Encouraging Participation UNIT 6: STRATEGIES, POLICIES AND PLANNING PREMISES Nature and Purpose of Strategies and Policies: The Key Function, The Guide, The Need for Operational Planning, The Effect on all Areas of Management - The Three Levels of Strategy: Corporate-Level Strategy : The values-based approach, The corporate portfolio approach, The BCG Matrix; Business-Level Strategy, Functional-Level Strategy - Strategic Planning: Characteristics of Strategic Planning, Significance of Strategic Planning, Limitations of Strategic Planning - Strategic Planning Process: Defining the Mission of the Organization,
  • 9. Drawing up Organizational Objectives, Assessing Organizational Resources, Risks and Opportunities, Formulating Strategy, Implementing Strategy, Monitoring and Adapting Strategic Plans; Strategic Planning Vs. Operational Planning - Competitive Analysis in Strategy Formulation: Environmental Assessment-Porter’s five competitive forces model, Organizational Assessment-SWOT Analysis - Major Kinds of Strategies and Policies: Growth, Finance, Organization, Personnel, Public relations, marketing - Porter’s Competitive Strategies: Overall Cost Leadership, Differentiation, Focus - Strategy Implementation: Carrying Out Strategic Plans – Technology, Human resources, Reward sys tems , Decision processes, Structure; Maintaining Strategic Control - Effective Implementation of Strategy: Communicating Strategies to all Key Decision-making Managers, Developing and Communicating Planning Premises, Developing an Appropriate Fit between Organizational Structure and Planning Needs, Ensuring that Action Plans Contribute to and Reflect Major Objectives and Strategies, Developing Contingency Strategies and Programs, Reviewing Strategies Regularly, Continuing to Emphasize Planning and Implementing Strategy, Creating a Proper Organizational Climate - Planning Premises: Planning Premises Vs. Future Expectations, Effective Premising – Selection of premises that affect the plans of the organization, Development of alternative premises for contingency planning, Verification of consistency of premises, Communication of the premises UNIT 7: MANAGERIAL DECISION MAKING Significance and Limitations of Rational Decision-making - Managers as Decision-makers: The Rational Model, Non-Rational Models – Satisficing model, Incr emental model, Garbage-can model - Decision-making Process: Identifying the Problem, Identifying Resources and Constraints, Generating Alternative Solutions, Evaluating Alternatives, Selecting an Alternative, Implementing the Decision, Monitoring the Decision - Types of Managerial Decisions: Programmed Decisions, Non-programmed Decisions - Decision-making under Certainty, Risk and Uncertainty: Decision-making under Certainty, Decision-making under Risk, Decision-making under Uncertainty, Risk analysis, Preference or utility theory - The Systems Approach to Decision-making: Management Information System, Decision Support System - Group Decision-making: Forms of Group Decision-making – Interacting groups, Delphi groups, Nominal groups -Decision-making Techniques: Marginal Analysis, Financial Analysis, Ratio Analysis, Break- even Analysis, Operation Research Techniques – Linear Pr ogramming, Queuing or waiting- line method, Game theory, Simulation, Decision-tree UNIT 8: FUNDAMENTALS OF ORGANIZING Definitions of Organizing - Benefits of Organizing - Traditional Perspectives on Organizing: Four principles from Fayol and Taylor’s theories of organizing: Unity of command, Well- defined hierarchy of authority, Authority at par with responsibility, Downward delegation of Authority and not Responsibility; Challenges to the Traditional View of Organizations – Bottom-up authority, Environmental complexity and unce-rtainty - Closed System Vs. Open System: Closed System view of Organizations, Open System view of Organizations – Daniel Katz and Robert Kahn’s Open-system characteristics, Developing an open system model - Formal Vs. Informal Organization - The Process of Organization: The Logic of Organizing, Some Misconceptions - Bases for Departmentation: Departmentation by Simple Numbers, by Time, by Process or Equipment - Choosing the Pattern of Departmentation.
  • 10. UNIT 9: STRATEGIC ORGANIZATION DESIGN Span of Management: Tall versus Flat Structure, Factors Determining an Effective Span – Trained subordinates, Clarity of delegation of authority, Clarity of plans, Use of objective standards, Rate of change, Communication techniques, Amount of personal contact needed, Organizational levels, Use of staff assistance, Supervision by others, Other factors - Authority Defined - Power:Bases of Power:Legitimate Power, Expert Power, Referent Power, Reward Power, Coercive Power - Line and Staff Relationships: Concept of Line and Staff, Functional Authority, Line and Staff Conflicts – Viewpoint of line m anagers, Viewpoint of staff managers, Nature of Line and Staff Relationship, Avoidance of Line and Staff Conflict - Centralization Versus Decentralization: Factors responsible for relative centralization and decentralization in an organization: History of an Organization, Availability of Competent Managers, Size of the Organization, Geographical Dispersion, Complexity of Tasks, Time Frame of Decisions, Importance of a Decision, Planning and Control Procedures, View of Subordinates, Environmental Influences - Delegation of Authority: Factors Affecting Delegation of Authority – The delegator’s aspect,The delegant’s aspect,The orga nizational aspect - Balance:The Key to Decentralization UNIT 10: STRATEGIC ORGANIZATION STRUCTURE Ensuring Understanding of Organizational Structure: Teaching the Nature of the Organizations’ Structure, recognizing the Importance of the Informal Organization and the Grapevine - Designing Organizational Structures: An Overview: Which Comes First: Strategy or Structure?, Factors Influencing Organization Design - Major Structural Alternatives: Advantages, Disadvantages and Uses of Functional Structure, Divisional Structure – Product divisions, Geographic divisions, Customer divisions, Hybrid Structure, Matrix Structure - Strategic Business Units UNIT 11: EFFECTIVE ORGANIZING AND ORGANIZATIONAL CULTURE Prerequisites for Effective Organizing - Avoiding Mistakes in Organizing by Planning: Planning for the Ideal, Modification for Human Factor, Advantages of Organization Planning - Avoiding Organizational Inflexibility: Signs of Inflexibility, Avoiding Inflexibility through Reorganization, The Need for Readjustment and Changes - Avoiding Conflict by Clarification: Organization Charts, Position Descriptions -Organizational Culture: Meaning, Significance and Characteristics of Organizational Culture, The organizational socialization process and steps - organizational Environment for Entrepreneuring and Intrapreneuring UNIT 12: HUMAN RESOURCE MANAGEMENTAND STAFFING Human Resource Management: An Overview :Human Resource Planning, Forecasting manpower demand, Forecasting manpower supply-Internal labor supply and External labor supply, Human resource actions, Staffing , Training and Development, Performance Appraisal, Compensation - Recruitment: Sources of Recruitment, The Recruitment Process - Selection: The Selection Process - Socialization Process of New Employees. UNIT 13: PERFORMANCEAPPRAISAL ANDCAREER STRATEGY Significance of Appraisal - Informal vs. Formal Appraisals - Performance Rating Methods: Graphic Rating Method, Behaviourally Anchored Rating Scale (BARS) - Criteria for Appraising Managers: Appraising Manages against Verifiable Objectives, Appraising
  • 11. Managers as Managers - Formulating Career Strategy: Preparation of a Personal Profile, Development of Long-range Personal and Professional Goals, Analysis of Environment Threats and Opportunities, Analysis of Personal Strengths and Weaknesses, Development of Strategic Career Alternative, Consistency Testing and Strategic Choices, Development of Short-range Career Objectives and Action Plans, Development of Contingency Plans, Implementation of a Career Plan, Monitoring Progress UNIT 14: ORGANIZATIONAL CHANGE AND ORGANIZATIONAL DEVELOPMENT Organizational Change: Factors that Lead to Organizational Change, Sources of Resistance to Change – Individual and Organizational resistance, Change P rocess - Planned Change through Organization Development: The objectives of OD - Organizational Development Process: Diagnosis, Intervention – Process consultation, Team building , Third-party intervention, Survey Feedback, Technostructural activities, Skill development, Organizational culture change; Evaluation - Approaches to Manager Development: On-the-Job Training, Off-the-Job Training - Organizational Conflict: Sources of Conflict, Managing Conflict UNIT 15: MANAGING AND THEHUMANFACTOR The Nature of People: Individual Difference, The Importance of Personal Dignity, Considering the Whole Person, Multiplicity of Roles - Behavioral Models: Edgar H. Schein’s Model of the Complex Person, Lyman Porter’s Model of Human Nature, McGregor’s Theory X and Theory Y, Three Managerial Models by Raymon E. Miles -Managerial Creativity: The Creative Process, Techniques to enhance Creativity; The Creative Manager UNIT 16: MOTIVATING EMPLOYEES FORJOB PERFORMANCE Definitions and Meaning of Motivation - Classification of Motivation Theories: Content Theories of Motivation – Maslow’s needs hierarchy theory, Herzberg’s two-factor theory, McClelland’s needs theory, Alderfer’s ERG theory; Process Theories of Motivation – Vroom’s expectancy theory, The Porter-Lawler model, Equity theory of J Stacy -Motivational Techniques: Rewards, Participation, Quality of Work Life (QWL), Job Enrichment - A Systems and Contingency Approach to Motivation UNIT 17: LEADERSHIP Definition and Meaning of Leadership -Key Elements of Leadership -Leadership Theories: Trait Theory of Leadership, Behavioral Theories, Likert’s Four Systems of Management, The Managerial Grid, Situational or Contingency Theories, Transformational Leadership Theory UNIT 18: MANAGING COMUNICATIONS Definitions of Communication - Significance of Communication in Organizations - Communication Process -Communication Flows in an Organization: Downward, Upward, Crosswise Communication - Barriers to Communication: Lack of Planning, Badly Expressed Messages, Faulty Translations, Unclarified Assumptions, Semantic Distortion, Loss by Transmission and Poor Retention, Communication Barriers in the International Environment, Inattention and Premature Evaluation, Impersonal Communication, Insufficient Adjustment
  • 12. Period, Information Overload, Lack of Trust in the Communicator, Other Communication Barriers - Gateways to Effective Communication: Interpersonal Trust, Effective Listening, Proper Feedback, Non-verbal Cues, Non-directive Counseling UNIT 19: THE CONTROLFUNCTION Planning and Controlling - Importance of Controlling: Coping with Uncertainty, Detecting Irregularities, Identifying Opportunities, Handling Complex Situations, Decentralizing Authority, Minimizing Costs - Levels of Control: Strategic Control, Tactical Control, Operational Control - Basic Control Process: Determining Areas to Control, Establishing Standards, Measuring Performance, Comparing Performance against Standards, Recognizing Good or Positive Performance, Taking Corrective Action when Necessary, Adjusting Standards and Measures when Necessary - Direct Control vs Preventive Control - Types of Control: Control Based on Timing, Cybernetic and Non-cybernetic control - Requirements for Effective Controls - The Certified Management Audit and Enterprise Self-Audit UNIT 20: CONTROL TECHNIQUES Major Control Systems: Managerial level – Types of cont rol system; Nature of timing – Types of control system - Financial Control: Financial Statements, Ratio Analysis - Budgetary Control: Responsibility Centers, Standard cost centers, Discretionary expense centers, Revenue centers Profit centers, Investment centers, Uses of responsibility centers - Quality Control: Quality Circles, Total Quality Management (TQM) - Inventory Control UNIT 21: PRODUCTIVITYAND OPERATIONS MANAGEMENT Production and Productivity - Productivity Problems and Measurement - Operations Research, Production & Operations Management: Operations Management and its Importance, Operations Research for Planning, Controlling and Improving Productivity, The essentials of operations research, operations research methodology - Some Operations Research Techniques: Linear Programming, Inventory Control, Significance of Inventory, Costs of Inventory, Economic Order Quantity, Just-in-time inventory system, Kanban, Distribution logistics, Time- event Networks, Value Engineering ,Work Simplification - Limitations of Operations Research: Magnitude of Computation, Gap between Managers and Operations Research, Lack of Quantification and Involvement of Qualitative Factors UNIT 22: MANAGEMENTINFORMATIONSYSTEMS Management Information: Meaning Attributes and Information needs of Managers - Components of an Information System: Hardware, Software, People, Data, Procedures - Types of Information Systems: Transaction Processing Systems, Office Automation Systems, Decision Support Systems, Executive Support Systems - Management Information Systems: Evolution of MIS, Computers and MIS, Advantages of MIS, Difference between MIS and DSS.
  • 13. CONTENTS Multiple Choice Questions 1 Unit 1 – Management: An Overview 1 Unit 2 – Evolution of Management Thought 3 Unit 3 – Social and Ethical Responsibilities of Management 5 Unit 4 – Fundamentals of Planning 7 Unit 5 – Management by Objectives 9 Unit 6 – Strategies, Policies and Planning Premises 11 Unit 7 – Managerial Decision Making 13 Unit 8 – Fundamentals of Organizing 15 Unit 9 – Strategic Organization Design 17 Unit 10 – Strategic Organization Structure 19 Unit 11 – Effective Organizing and Organizational Culture 21 Unit 12 – Human Resource Management and Staffing 23 Unit 13 – Performance Appraisaland Career Strategy 25 Unit 14 – Organizational Change and Organization Developmen t 27 Unit 15 – Managing and the Human Factor 29 Unit 16 – Motivating Employees for Job Performance 31 Unit 17 – Leadership 33 Unit 18 – Managing Communications 35 Unit 19 – The Control Function 37 Unit 20 – Control Techniques 39 Unit 21 – Productivity and Operations Management 41 Unit 22 – Managerment Information Systems 43 Multiple Choice – Answers and Explanations 45
  • 14. MULTIPLE CHOICE QUESTIONS UNIT 1 – MANAGEMENT: AN OVERVIEW In a management quiz, the quiz master asked as to who has defined management as “the process of designing and maintaining an environment in which individuals, working together in groups, efficiently accomplish selected aims.” The correct ans wer is__________________. a. Harold Koontz b. Heinz Weihrich c. Henry Mintzberz d. a & b e. a & c On observing a drop in its first quarter earnings, a manager in a mobile manufacturing industry wanted employees to pool ideas for innovation. The employees came up with an idea of developing new apps in mobile where it had to get the permission from government officials to import some of the chips from a foreign company. Then the manager assured themto get the same talking to government officials. Name the decisional roles played by the manager. a. Entrepreneurial role b. Disturbance handler c. Resource allocator d. a & c e. a & b Ritesh had a plan to start cab facility dealing with wide range of cars from normal to luxuries. His objective was to provide the cheapest cab services with maximum comfort for the passengers. He then selected the cities to locate offices and recruited people to manage those offices. He then delegated the duties and responsibilities to the staff and took charge of supervising the overall branches. What are the different functions Ritesh has performed? a. Planning &Organizing b. Staffing c. Leading d. Controlling e. All the above Karthik as a supervisor has to manage the day to day tasks in a manufacturing division. He also has to check the performance of the staff and ensure implementation of operational plans according to the organizational requirements. Identify the managerial level that Karthik fits into. a. Top-level managers b. Middle-level managers c. First-level managers d. Second-level managers e. None of the above The top-level managers spend time in planning and organizing and first-level supervisors spend time in communication. In addition to these there are many factors. Name the factor which is similar at all the hierarchical levels in management. a. Controlling b. Supervising c. Operating d. All the above e. None of the above Karan is Head-Accounts manager in a construction company. He co-ordinates with different site supervisors and accumulates all the details of the accounts from different construction sites and branches periodically, consolidates and reports the details to his superior. He prepares the balance sheet and gets it audited by internal and external auditors as well. What are the different skills that are demonstrated by Karan? 1
  • 15. Principles of Management a. Technical skills b. Human skills c. Conceptual and Design skills d. a & b e. a & c Manisha is a team leader in a software company. In order to meet her project deadlines, she conducts meetings regularly, checks the progress status and motivates her team members accordingly. She also helps them to tackle the problems encountered in the process. Identify the skill exhibited by her to get her work done. a. Technical skills b. Human skills c. Conceptual skills d. Design skills e. All the above A marketing company proposed to combine its marketing and sales department to foster coordination and improve sales. This decision has changed the job profiles of all the sales and marketing team and the HR manager had to communicate their new profiles. Accordingly, changes were made in the organizational structure and hierarchy in the marketing department. Identify the management approach applied in the case. a. The Interpersonal Behavior Approach b. The Empirical or Case Approach c. The Operational Approach d. The McKinsey’s 7-S Framework e. The Mathematical or “Management Science” approach When a supermarket found that long queues are obstructing the billing staff to concentrate, it has planned to increase billing stations to reduce the crowd. The management collected information on number of customers arriving at each station at each branch and used statistical methods to come out with number of billing stations to be increased in each branch. Name the management approach used in the case in decision making. a. The Interpersonal Behavior Approach b. The Management Science Approach c. The Cooperative Social Systems Approach d. The Decision Theory Approach e. None of the above A Supervisor observed that an employee was coming late even after several reminders and as per the company’s rule he needs to be dismissed. But on enquiry, the manager found that his mother was seriously ill. He understood the circumstances and counseled him to make alternate arrangements. Find out the management approach that manager has used. a. The Contingency or situational Approach b. The Operational Approach c. The Socio-technical Systems Approach d. The Empirical or Case Approach None of the above 2
  • 16. Multiple Choice Questions UNIT 2 – EVOLUTION OF MANAGEMENT THOUGHT Management students in a class were being taught about the evolution of management. A student in the class raised doubt as to who was the first to contribute to the study of management concept. Identify the person fromthe following: a. Charles Babbage b. Robert Owen c. Henry Robinson Towne d. Yale and Towne e. Andrew Ure In a quiz competition at a Management School, Quiz master asked to name the person who is widely known as the ‘Father of modern computing” and also a pi oneer in the field of management. He is the person who invented the world’s fir st mechanical calculator and the “analytical engine”. The student’s answer was a. Henry Laurence Gantt b. Frank and Lillian Gilbreth c. Andrew Ure and Charles Dupin d. Robert Owen e. Charles Babbage Harry was sharing his job experiences with one of his friends and was explaining how their management deals with every issue in a scientific manner and provides the required support to the employees. He was reminded of F.W. Taylor who proposed the scientific approach for the first time. Immediately his friend argued that the first thought of scientific management was not from Taylor and was by someone who inspired Taylor. Identify the person from the following. a. Henry Robinson Towne b. Robert Owen c. Charles Babbage d. Mary e. None of the above A supervisor in a company doubtful of completing the production s chedule on time wanted to plan a strategy that could assure completion. He then decided to check and reduce the time taken to complete each task without compromising with quality to set a standard. He also planned to reward the worker who meets the established standards of performance set by the management. Name the school of thought that inspired him a. Scientific Management b. Administrative theory c. Bureaucratic Management d. Quantitative approach e. All the above In an Operation Management classroom, the professor was explaining about the latest Program Evaluation and Review Techniques (PERT) that is used as production control technique. A student in the classroom had a doubt about the earlier techniques of control before the PERT model was introduced. Pick up the answer from the following. a. Micromotion study b. Time-and-motion study c. Gantt Chart d. Piece-rate incentive system e. Gantt bars A parent interested to check their children’s progress, went to their school one day. She had to comply with all the rules and regulations of the school to meet the concerned authorities and there was a line of hierarchy before she met the principal to clarify her doubts. Name the kind of administration followed by the school. a. Scientific management b. Bureaucratic management c. Social management d. Administrative management None of the above 3
  • 17. Principles of Management Today, organizations are searching for multitasking individuals to meet the challenges of current scenario and to encourage employees perform a variety of jobs in organization. There also exists a conventional theory that strongly supports this modern practice. Name the person who proposed the conventional theory from the following: Mary Parker Follet: Focusing on Group influences Elton Mayo: Focusing on Human Relations Abraham Maslow: Focusing on Human Needs Douglas McGregor Chris Argyris: Matching Human and Organizational Development An organization in its problem diagnosis found that its current manual data management system was not effective and need an effective one. They searched for the latest technology and implemented it where data collection, analysis and reports were easily accessible and manageable to the managers. What is that management technique we are talking about? a. Management Information Systems b. Operations Management c. Management Science d. Scientific Management e. Management science theory Consequent to increased employee turnover, an organization decided to provide job security and welfare measures for its employees. In addition, to reduce their boredom and to offer promotions, training sessions were planned. This has reduced the labor turnover and absenteeism and increased productivity. Name the theory applied in the case. a. Theory Y b. Systems theory c. Administrative theory d. Theory Z e. Contingency theory Gautam, a general manager in a manufacturing company tried to implement a strategy that was successful in solving a problem. He tried to implement the same strategy to solve a similar problem in another department but failed. From this experience he has learnt that there is no single principle to manage all the situations in an organization. Extract the important theory that he has learnt in the case from the following. a. Systems Theory b. Contingency Theory c. Theory A d. Theory Z Theory X 4
  • 18. Multiple Choice Questions UNIT 3 – SOCIAL AND ETHICAL RESPONSIBILITIES OF MANA GEMENT The K. C. Mahindra Education Trust, a part of Mahindra & Mahindra started a number of education plans to change the lives of worthy students. More than 7.5 crore were sanctioned in the form of grants, loans and scholarships to encourage education in India. It has about 3,300 children in its project called ‘The Nanhi Kali’. To reach the n eglected children in rural areas, it has plans to increase the number to 10,000 in the next two years. Name the responsibility it is taking up. a. Political responsibility b. Social responsibility c. Management responsibility d. Organization responsibility e. Economical responsibility Vodafone, as a part of its corporate social responsibility is trying to improve the efficiency in energy consumption and to recycle its network equipment waste by 95%. Thus, it guarantees to reduce their carbon dioxide emissions by 2020. In what way does this contribute to society? a. Avoiding intervention by government b. Protecting shareholder interests c. New avenues to create profits d. Balance of responsibility and power e. None of the above Haagen-Dazs is an ice cream brand from New York. A micro-blog was created by this company to enlighten tweeters about the significance of honeybees.Through its “save the honeybee” campaign, it created awareness on the consequences ofreducing bee population.It informed that decline in bee population could reduce the global production and disappearance of several plant species.It also alerted that when bees die out, Haagen-Dazs may just go out of business.What kind of social responsibility is this? a. Endeavor to find new solutions b. Best use of resources of a business c. Favorable public image d. Balance of responsibility and power e. Change in public expectations McDonald’s sets its own standards on how suppliers have to take care of its employees. It also started a program called “Flagship Farms Initiative (FFI)” throug h which it promotes the creative and ethical farming practices. All these depict the social responsible practices of McDonald’s in its supply chain strategy. Suppliers being one among the six major interest groups for organizations, what are the other groups interested? a. Shareholders b. Employees c. Government d. Society e. All the above Infosys Technologies has a CSR foundation. It conducts training programmes and promotes computer education in village schools. Name the category of social responsiveness Infosys has demonstrated. a. Fund-raising b. Recycling c. Direct corporate investment d. Contributions e. Quality of work life Hindustan Zinc Limited reviews its monitoring systems for environmental safety at its mining units. It maintains eco-friendly environment by building control plants for dust emission, mine dumps for solid waste disposal, waste water treatment systems and more. Which category of social responsiveness are we talking about? a. Contributions b. Recycling c. Attention to consumers d. Pollution control Valuing diversity 5
  • 19. Principles of Management Apple Inc. is the largest US-based technology firmknown for its innovative products. At the same time, it is also known for maintaining secrecy to an extent that secrecy has become one of the key fundamentals of its corporate culture. In 2009, it was blamed for not disclosing the crucial information regarding CEO Steve Job’s health to its shareholders. It also had to face criticism for maintaining the information as a secret. This depicts the organization’s ____________. a. Social responsibility b. Social audits c. Code of conduct d. Social adaptability e. Managerial ethics Exxon Mobil’s main products are petroleum and gasoline owning refineries to drill and pump crude oil all across the world. In 2007, Exxon Mobil was blamed for misguiding the public claiming that it was concerned about the global warming and taking measures to reduce their carbon footprint through commercial releases. They have removed these commercials immediately when questions arose from the advertisements. What is the factor that influenced the ethical behavior in the case? a. Structural variable b. Issue intensity c. Organization’s culture d. Individualcharacteristics e. None of the above Clinical trials are essential for the development of medical science and advancement in therapeutics. But protecting the human rights and human safety plays a primary role in clinical trials. In India, the regulatory agency that oversees and approves clinical trials is the CDSCO (Central Drugs Standard Control Organization). Who is responsible for overseeing their activities? a. Ethics Committee b. Ethics audits c. Code of ethics d. Ethics training e. None of the above Adidas always looks out for innovative ways to connect directly with its suppliers/ workers. In 2012, the Adidas group wanted to test mobile phone communication at one of the company’s main footwear suppliers in Indonesia and started sending SMS to its workers. Through which channel, is Adidas trying to get connected with its suppliers? a. Ethics training b. Ethics hotline c. Ethics committee d. Ethics audit None of the above 6
  • 20. Multiple Choice Questions UNIT 4 – FUNDAMENTALS OF PLANNING The CEO and Board members of a Food Products company set goals to increase its sales by implementing new flavors of chips from the next quarter. Based on these goals, the middle level managers started collecting the customer tastes through market research team and subsequently decided flavors with the help of top level executives. Instructions were given to the operating supervisors to make necessary arrangement for the production of new flavored chips. Which nature of planning is depicted in this case? a. Planning is goal-oriented b. Planning is all-pervasive c. Planning is forward-looking d. Planning is an integrated process All the above Amit had a plan to supply building materials and was searching for the right place. He located a place where a group of real estate ventures were approved by the government. He chose to start his business there anticipating huge demand for the material supplies when the ventures get started. Where can we fit the situation in significance of planning? Planning provides sense of direction Planning increases organizational effectiveness Planning provides efficiency in operations Planning offsets uncertainty and risk None of the above An FMCG organization’s goal was to exceed the sales target of the last quarter. Accordingly a meeting was conducted with all the department heads. The marketing manager was assigned the responsibility to ensure timely supply of goods to the sales executives coordinating with the production manager. In turn production manager was required to produce goods on time coordinating with finance department. This case depicts the significance of planning in_______. a. Facilitating delegation b. Ensuring bettercoordination c. Encouraging innovation and creativity d. Facilitating control e. Providing efficiency in operations To gain competitive advantage through quality products, top management of an Electric appliance organization decided to implement Total Quality Management in the entire organizational operations and processes within next five years. The Board of directors, CEO and heads of the departments passed a resolution to implement the same. Name the type of plan discussed in the case. a. Strategic plans b. Tactical plans c. Operational plans d. Single-use plans e. Standing plans After Chanda Kochhar joined ICICI Bank, organization started rethinking. Organization decided not to hire former employees, not to sack employees even during recession, underperforming employees were declared ineligible to increments, decided to continue giving medical support till 60 years of age even after leaving the organization to boost up the morale and improve the performance of employees. Which aspect of the organization has got changed? a. Budgets b. Policies c. Programs d. Procedures Projects 7
  • 21. Principles of Management An organization planned to open a branch in foreign country within two years. It has instructed its executives to survey market potentials in the first year and select the location and organize resources the next year. Name the type of plan based on the time-frame. a. Short-term plan b. Long-term plans c. Intermediate-term plan d. Very short-term plan e. None of the above Exploring the craze for eco-friendly plastic vessels, Nidhi wanted to start one such manufacturing unit. Her objective was to deliver goods as per customers’ tastes at lowest price. She has selected three different places to establish the plant. While one location was near to the finished goods supplies, the other location was near to raw material supply. The third location was suitable for the availability of cheap labor. She started analyzing all the three alternatives to choose the best one. Where exactly she is in the process of planning? a. Analyzing opportunities b. Determining planning premises c. Establishing objectives d. Evaluating available alternatives Identifying alternatives A Bank manager at a branch decided to improve its operations to foster the process and reduce the service time. He collected suggestions of customers through suggestion box and questionnaires. He requested his employees to suggest effective ways to improve the performance of bank during meeting. Consequently,he came up with an innovative and effective plan that could achieve his objective. Identify the systems approach that helped him in effective planning. a. Management Information System b. An Open Systems Approach c. Planning Premises d. Closed systems approach e. None of the above Sirish planned to open a computer training institute to earn lump sum by offering latest technologies expected to be in the market shortly . In order to start the program, he started collecting information on the technology and best personnel available to teach those technologies. It took him almost six months to complete the activity. By then many institutes emerged in the area competing with him. Name the planning limitation that shattered his expectations. a. Lack of accurate information b. Inflexibility c. Resistance to change d. Environmental constraints e. Time consuming process In a manufacturing company, the supervisor passed a circular asking employee to work two hours extra a day to complete the production schedule by next week. Workers refused to do so as it was against their agreement with the management and the change was without their consent. What do you call this situation in an organization? a. False sense of security b. Reluctance to establish goals c. Lack of ability and commitment d. Resistance to change Environmental constraints 8
  • 22. Multiple Choice Questions UNIT 5 – MANAGEMENT BY OBJECTIVES Karan, a production manager of AGK group attended a meeting with CEO along with the heads of other departments to discuss targets for the coming assessment year. She was worried about the target assigned as the productivity level of employees was continuously falling. She then called her team to convey the target and invite suggestions and set individual targets. They decided to follow up every week with feedback. Finally they could meet the target. Identify the type of management followed by her. a. Management by objectives b. Managementby exception c. Organization by objectives d. Organization by Exception e. Management by Goals Karthik is a CEO of Creative Minds Ad agency. One day he called his manager and informed about his goals for the next year and asked him to set objectives and means to achieve those objectives discussing with the employees. Name the hierarchy of objectives. a. Top-down approach b. Vertical approach c. Bottom-up approach d. Diagonal approach e. None of the above Organizations would first develop overall goals, next establish specific goals for various departments, subunits and individuals. Then it would formulate action plans, implement and maintain self- control through periodical reviews and appraises. These statements are explaining about the __________. a. MBO characteristics b. MBO process c. Benefits of MBO d. Limitations of MBO e. Concepts of MBO In MBO program, employees are encouraged to set their own objectives and asked to show __________________ to achieve the goals. a. Satisfaction b. Motivation c. Management d. Commitment e. Clarity The marketing VP of XYZ company observed that national sales went down by 9% for the last quarter. When the president demanded a remedy, fearing to lose his job, he instantly increased national advertising which improved sales for that moment. But while doing so he ignored its effect on the long term. Identify the area focused by the VP in the case. a. Failure to teach MBO philosophy b. Inflexibility c. Failure to give guidelines to goal setters d. Difficulty in goal-setting e. Emphasis on short-term goals Mr. Bansal, manager of an online business first set organizational goals discussing with his subordinates and employees. Later, he set goals for individuals in each department and formulated action plans. He extended all his support required by his employees during the process. He gave themtime for six months and reviewed the work on the last day. But to his surprise, he saw some gaps in the performances. If followed, which step could have helped him in making his MBO process effective? a. Top management support b. Training for MBO c. Formulating clear objectives d. Encouraging participation Effective feedback 9
  • 23. Principles of Management Bhargav, working as head of a super market was given a target to increase the number of customers by his superior under MBO process. On the day of performance review, his superior found that there was only a marginal increase as against the set target. The superior being dissatisfied, asked Bhargav to give an explanation and cut his incentives for that quarter. What aspect of the MBO helped supervisor in taking the action? a. Basis for organizational change b. Encouragement of Personal commitment c. Development of effective controls d. Better managing e. Clarity in organizational action Manish, a team leader in a software company first set individual goals to all his teammembers. He then started reviewing the performance periodically. After two weeks, he found that one of his team members Mr. Nisheeth was not committed due to which the overall team’s performance was lagging behind. Mr. Manish could overcome this situation by_______. a. Encouraging participation b. Training for MBO c. Formulating clear objectives d. Effective feedback e. None of the above Karthik has joined newly as an executive in a multinational company. He was asked to prepare list of objectives along with his subordinates for the next assessment year. He was surprised as to why his subordinates should be included in setting objectives as his earlier boss never asked himto do so. On inquiring his senior manager, he was told that organization practices MBO. Identify the limitation factor of MBO that confused Karthik in the new office. a. Emphasizing on short-term goals b. Failing to teach MBO philosophy c. Inflexibility d. Failing to teach goal setting e. None of the above The MBO process should receive continuous support from__________managers forits successful implementation. a. Top-level b. Middle-level c. Low-level d. First-level Last-level 10
  • 24. Multiple Choice Questions UNIT 6 – STRATEGIES, POLICIES AND PLANNING PREMISES Mary, CEO of a confectionery planned to break into a new overseas market. She started developing relationships with overseas suppliers, identified network of retail outlets, conducted market research to identify consumer needs and finally found location for overseas sales team head quarter. All these activities reflect the organizations’________. a. Policy b. Goal c. Guidelines d. Strategy e. All the above A woman shoe company set its mission statement as “we put shoes on the feet of women who are style- conscious while insisting on health and ease”. Acc ordingly, the marketing manager suggested advertising the company’s shoes in high-fashion health stores for the first time. How could the marketing manager come up with such an idea? a. Giving direction to organizing b. Giving direction to planning c. Giving direction to staffing d. Giving directions to operationalplanning e. None of the above Nestle company introduced its products in Pakistan with full advertising on all media in 2012. It involved media, advertising and campaigns for promotions. The company’s financials for the first quarter ending March 31, 2012 showed a profit of Rs.1.66 billion. They evaluated their business portfolios (Mineral water, Milkpack, Noodles and chocolates/juices) in two dimensions i.e. market growth rate and relative market share. The results were displayed as follows: Product Name SBU Market Growth Relative Rate Market Share Mineral Water Star 85% 89% Milkpack Cash cow 55% 65% Noodles Question Mark 75% 44% Chocolates/Juices Dog 40% 38% Identify the corporate-level strategy followed by Nestle. a. The value-based approach b. The strategy portfolio approach c. The corporate portfolio approach d. All the above e. None of the above Reliance Industries Limited manages textile fabrics and a range of petrochemical products. For each product group, the nature of market in terms of customers, competition, and marketing channel is differentiated. It plans different strategies for its different product groups. Each Strategic Business Unit (SBU) builds its own strategies to utilize its resources effectively. Name the organizational strategy used by the Reliance Industries. a. Corporate-level strategy b. Functional-level strategy c. Department-level strategy d. Business-level strategy e. All the above Nokia came to India to exploit the diverse market, especially the unexploited market in rural India. Its mission was to provide the customer with durable and low cost cell phones. It formulated marketing objectives to achieve its goals by conducting SWOT analysis. After analyzing the resources and environment to operate, its major strategy was to continue as No.1 position in India’s rural and urban market. It adopted low cost strategy for rural India and differentiation strategy for urban India. Name the process that Nokia adopted to capture the Indian market. 11
  • 25. Principles of Management a. Strategic planning process b. Administration process c. Crisis management process d. Competitive process e. None of the above A quality inspector in a manufacturing organization proposed a strategy to adopt latest safety measures to which the management agreed. Accordingly, he wanted to inform his workers about the latest safety measurement tools procured by the organization. He sent a circular to the manufacturing unit specifying the list of tools and procedures to use in writing. To that effect, he also sent some pictures to display at the unit. Name the strategy that is used by the Quality inspector. a. Communication strategy b. Marketing strategy c. Sales strategy d. Project strategy e. None of the above A service business unit initiated a strategy to retain its existing customers and increase sales. It changed the compensation system and the targets for its managers. It discarded the old practice of measuring senior managers by revenue and margin targets and fixed 20 percent of their compensation to achieve the retention goals. It also offered other bonus packages. These initiatives taken by the company encouraged managers to make the strategy successful. Identify the factor that helped the organization in its strategy implementation. a. Technology b. Decision processes c. Structure d. Maintaining strategic control e. Reward systems Goodwin Company manufactures and sells domestic cleaning products. Its target was to earn a profit of billion dollars in household business in one year. But due to new entry in to the bus iness, it was able to earn only half a billion dollars through its business in household. Margerett, the sales manager quickly observed that there was huge potential of generating profits in retail market and decided to shift the focus to contract packaging and distribution in industrial and automotive cleaning markets. This helped her to meet the target of the organization. Name the strategy that helped her to achieve the target. a. Communication strategy b. Implementation strategy c. Contingency Strategy d. All the above e. None of the above Bhargav, a General Manager was developing plans to enter overseas market that was set as a strategic vision by the company. After a SWOT analysis, he chose a specific country but found that several competitors with similar products have entered that market recently. Bhargav after discussing with the Superiors decided to choose the promising opportunity of introducing a new product. He developed plans to challenge the competitors through marketing efforts to introduce the new product to challenge competitors successfully. Identify the element of planning that helped Bhargav to meet the competition. a. Organization premises b. Planning premises c. System premises d. Market premises Produce premises In the above case 59, Bhargav took the help of his subordinates in exploring the overseas market and coming up with a new challenge to introduce a new product in place of similar products. Name the factor that helped him to effectively implement the strategy. Developing an appropriate fit between organizational structure and planning needs Creating a proper organizational climate Reviewing strategies regularly Ensuring action plans to reflect major objectives and strategies Maintaining strategic control 12
  • 26. Multiple Choice Questions UNIT 7 – MANAGERIAL DECISION MAKING Violet Jones is a manager at the fast food restaurant. She got a mail from the headquarters to increase her monthly profits. She determined to select the best path to save money and decided to cut costs in other ways than eliminating employees. She chose other alternatives such as finding cheaper vendors, reducing store hours, changing menu options, etc. Violet evaluated all alternatives and after much consideration, decided to submit her recommendations to management with the hope that it will increase store profits. Name the decision-making process described in the case. a. Rational decision-making b. Irrational decision-making c. Garbage-can decision-making d. Satisficing decision-making e. Incremental decision-making Hari went to purchase a best model car available in Hyderabad that could suit his budget, give him all comforts, have the maximum seating capacity and belongs to a top brand. At the showroom, he found that there are no models that can meet all his demands and finally he had to choose the one that best suited his budget. Identify the decision-making model used by Hari. a. Rational model b. Incremental model c. Garbage-can model d. Satisficing model e. Programmed decision model From several decades, customers of fast food restaurants were concerned with the taste and the price of food rather their health. In 2005, Mc Donald encountered an unusual problem when customers showed growing concern with regard to foods high in fat and calories. In response, McDonald’s started offering healthy food by substituting apple slices in Happy meals to French fries and stopped using trans-fats. Identify the decision making approach followed by McDonald in the above case. a. Programmed decision-making b. Non-programmed decision-making c. Incremental decision-making d. Garbage-can decision-making e. Bounded decision-making To introduce a new caffeine-free cola, the vice-president of marketing for a Cola company was considering several plans. One plan was expected to cost him 10 crores while the other to cost only 5 crores. As per his estimations there was an equal chance of acceptance of the new cola both the ways and profits were also marginal. He conducted expected monetary value analys is to decide about the type of plan to be implemented. Determine the decision-making category applied in the above case. a. Decision-making under certainty b. Decision-making under risk c. Decision-making under uncertainty d. Decision-making under stability e. Decision-making using preference or utility theory Prateek, a manager of a production floor that has different lines of production wanted to keep track of labor costs. He started collecting and comparing the pay roll reports to the budget set by the organization in order to implement corrective actions, if necessary. These reports were required every week. He observed that the time taken to complete this process is eating up most of his valuable time and wanted to adapt a system that could help him with speed, precision and economy in carrying out this function effectively. Kindly help him with the right suggestion from the following alternatives. a. Management Expert System b. Transaction Support System c. Management Decision System d. None of the above Management Information System 13
  • 27. Principles of Management ABZ Engineering company is committed to a specific decision making approach. Whenever the company calls for interviews, eight to ten employees are involved in interviewing every job candidate. Also when the company has to take major decisions, it forms a committee involving employees from all the levels of management to address the issue. Name the approach followed by the company. a. Group decision-making b. Expert decision-making c. Operational decision-making d. Individual decision-making e. Strategic decision-making Vice President of a large manufacturing company planned to replace technology for manufacturing a new product line. He selected members of the organization, plant managers, executives, consumers and some renowned experts in the field to discuss on the matter. He then collected the opinions of all the experts to calculate average and take the decision based on the results. Name the technique used by the Vice President to take the decision in the above case. a. Nominal group technique b. Interacting group technique c. Delphi group technique d. Break-even analysis technique e. Ratio analysis technique After renovating the restaurant, a restaurant manager observed that the customer arrivals have increased and some of the customers are kept waiting for long time to serve. He decided to increase the number of servers. He then took the help of an operation technique to suit the number of servers to the increased customer demand. Identify the technique used by the restaurant manager fromthe list given below: a. Linear programming b. Game theory c. Simulation d. Queuing theory e. Replacement theory A manufacturer wanted to sell a product. He had two options before him. The product can be either made inside the factory or purchased from outside. While producing inside would require using some of the scarce resources, procuring from outside would not. To achieve his goal of minimizing the total cost, he used a technique to come out with the best solution. He was successful by adopting such technique. Name the technique that helped him in making the best choice. a. Delphi group technique b. Simulation theory c. Linear programming technique d. Critical path method e. Ratio analysis technique Neeta was interested to invest in a company that can assure good returns. She selected some companies to test their past performances before investing. She started calculating the long term solvency, fixed assets to long term and debt percentages of all the firms and then compared their findings. She then selected a company whose financial condition was stable and invested. Name the analysis used by Neeta in this case. a. Ratio analysis b. Marginal analysis c. Financial analysis d. Comparative analysis Operations analysis 14
  • 28. Multiple Choice Questions UNIT 8 – FUNDAMENTALS OF ORGANIZING A Construction company manager is responsible for ensuring the project completion on time. For this, he has to appoint the best subcontractors and see that building materials reach the project place within the stipulated time and ensure proper utilization of resources and eliminate duplication of efforts. Identify the function of management the manager is performing in the construction company. a. Planning b. Marketing c. Organizing d. Controlling e. Staffing Karthik, a floor manager in a production department who has recently joined, examined the floor management at the manufacturing plant. He insisted the supervisor to display the flow of work activities being carried out at each floor. He conducted a meeting and informed the workers about the channel of communication to approach for any clarification. These efforts of the manager have brought improvement in the productivity of the workers. Identify the aspect of organizing that is explained in the case. a. Benefits of organizing b. Limitations of organizing c. Traditional perspectives on organizing d. Formal organization e. Informal organization A furniture manufacturer got an order for 300 lecture stands. He has five workers to do the job. There are two ways to complete this order. One is every worker can be asked to manufacture 60 lecture stands. The other method is to distribute different parts of the lecture stands, like top board, center support, assembling and polishing individually to different workers. The manufacturer strongly believes in a management principle and has adopted the second method. Name the principle the manufacturer believed in. a. Chain of command b. Span of control c. Hierarchy of authority d. Delegation of authority e. Division of labor An electronics supplies owner who observed downtrend in his sales volumes, immediately planned to collect feedback from his customers on the parameters of quality and the expected changes in the products. He then started supplying the information to the manufacturers to update their products as per the customer’s requirements. This approach helped not only the shop owner to improve his sales but also enhanced the quality of manufacturers through innovations. Name the approach that helped both the owner and the organization to increase their sales. a. Closed system b. Open system c. Stable system d. Unstable system e. None of the above Get-well, a super specialty hospital is located in an industrial Area. It has 160 beds capacity. It has 40 duty medical officers and 300 Para medical staff. It recently started 24 hours medical, surgical and cardiac emergency services. The team of medical officers, consultants and para medical staff serves the patients throughout the day by segregating themselves into three shifts. It makes its services available to customers round-the-clock. What form of departmentation is the hospital following? a. Departmentation by numbers b. Departmentation by simple numbers c. Departmentation by time d. Departmentationby process Departmentation by equipment 15
  • 29. Principles of Management A soap manufacturing company on being successful in achieving its target of highest market sh are in national markets started manufacturing cleaning products for domestic and industrial purpose. The General Manager-operations now is facing difficulty in managing the three lines of products due to increase in the volume of operations. Now the organization is planning to reduce the burden of its operations manager through departmentation. Identify the best suitable deparmentation for the organization. a. Departmentation by products b. Departmentationby process c. Departmentation by customers d. Departmentation by territories e. Departmentation by functions A Corporate bank whose services are spread across territorial regions has now planned to further provide its services by offering separate departments for commercial loans, installment loans, savings accounts, and checking accounts. The bank has planned to departmentalize its services based on ______________. a. Functions b. Territory c. Process d. Products e. Equipment 3M, an innovative company is a leading supplier for electronic products and protective products. At 3M, everyone is welcome to participate in innovation irrespective of their levels in the management. It offers best challenges to innovators who can convince themselves first and able to convince several levels of management about the value of those ideas. Name the approach followed by 3M in innovation. a. Top-down approach b. Bottom-up approach c. Diagonal approach d. Cross sectionalapproach e. Virtual approach Departmentation by ____________________ is a type of departmentalization which is widely used in manufacturing organizations where people and material are brought together to carry out a particular operation. It provides the benefit of specialization and facilitates the optimum use of resources. a. Process or equipment b. Product c. Territory d. Function e. Time Sphoorthy and Sneha, working for a company, became friends through regular meetings at lunch time in the canteen. Both of them are working under different teams of the same project. When Sphoorthy was in trouble to complete her task, Sneha helped spontaneously with her expertise and experience. What type of organization has helped Sphoorthy in overcoming her problem in the company? a. Formal organization b. Stable organization c. Flexible organization d. Informal organization Closed organization 16
  • 30. Multiple Choice Questions UNIT 9 – STRATEGIC ORGANIZATION DESIGN Ritesh is a sales executive at one of the branches of Machine Tools Supply Company. One day, his sales representatives reported a customer complaint regarding the quality of the tools. Ritesh forwarded the complaint to his manager and manager in turn passed it to the central manager. Ritesh has been following up since then. But It was almost a fortnight from the day the complaint was registered and there was no solution offered and the customer returned the tools being dissatisfied by the delay. What aspect of management was responsible for the delay in the case discussed above? a. Tall structure b. Flat structure c. Broad structure d. Wide structure e. Large structure Muthuraman, started a Steel Manufacturing company and was successful in gaining market share in his state. With the success, he started branches in other states and retained the power of decision - making with him. Slowly branches started improving sales and that demanded most of his time for decision making. To reduce the overburden, he gave some guidelines on decision-making to the branch manager and asked them to take charge. Name the factor of effective s pan of management Muthuraman has planned in the case. a. Use of objective standards b. Rate of change c. Clarity of delegation of authority d. Clarity of plans e. Trained subordinates Kannan has started a BPO with a staff of 50 numbers. He used to take care of sales, marketing, accounting, HR and IT manager responsibilities all alone. When the company started progressing, number of staff got increased to 300 and he was unable to manage the financial aspects of the company. He then decided to appoint an accountant to take care of company accounts. Which factor among the effective span of management helped Kannan to reduce his burden. a. Organizational levels b. Amount ofpersonalcontactneeded c. Use of staff assistance d. Communication techniques e. Trained subordinates Srinivas is working as VP for an Automobile Sales company. He sets targets to his sales managers of different branches. In terms of growing need of demand, he informed them that those who meet the sales target would be given incentives as a percentage over the generated profits and those who do not meet would be demoted. Name the power that the VP used in granting incentives to successful managers. a. Expert power b. Legitimate power c. Referent power d. Reward power e. Coercive power In case no.84, name the power used by the VP to demote the unsuccessfulmanagers. a. Legitimate power b. Reward power c. Coercive power d. Expert power e. Referent power Caterpillar Inc., an American corporation designs, manufactures and sells machinery and engines. At Caterpillar Inc., price related decisions were made only at the corporate headquarters. When a sales representative working in Africa wanted to give a discount on a product, he had to check with headquarters. Headquarters took more than a week’s time to check the markets for information to take effective decision. The delay in decision making made them to lose their customers. What type of management was Caterpillar following? 17
  • 31. Principles of Management a. Centralized b. Decentralized c. Flat d. Tall e. Informal Home Store is a retail giant with 2,000 stores across India. At each store, managers were given responsibility to manage a team of sales and meet the sales target. They were also responsible to handle complaints and resolve the problems of customers. To facilitate quick decision making, managers were also given authority to act autonomously. Name the type of authority the management is following? a. Centralized b. Functional c. Formal d. Informal e. Decentralized Preethi, a HR manager in a multinational company was deeply engaged in a thought on how to solve the problem of an employee. Her personal assistant observed the intensity of the problem and recommended some alternative solutions to her. Though the solutions were feasible and acceptable, Preethi refused her recommendations and shouted at her assistant not to involve in her job. Name the factor of line and staff conflict that affected Preethi. a. Viewpoint of line managers b. Lack of accountability c. Encroachment of line authority d. Dilution of authority e. Theoretical bias Charan who joined as an incharge for billing counters of big super market was promoted as supervisor looking at his relationship maintenance and control competencies. Cons equently, the branch manager was asked by the management to explain him the job profile of supervisor and help him in executing his tasks. Subsequently, whenever Charan approached the manager, he simply refused to help him fearing that Charan may excel in his job and become a good contender for him. What factor of delegation is seen affected in the case. a. Love for authority b. Fear ofsubordinates’advancement c. Fear of exposure d. Attitude towards subordinates e. Personality traits and experience of the superior Continuing the case no.89, for several months Charan managed to work without proper guidance from the manager. Finally, unable to sustain, he submitted his resignation to the management. Identify the factor that forced Charan to resign. a. Lack of information and resources b. Fear of criticism c. Lack of self-confidence d. Absence of rewards and incentives Fear of exposure 18
  • 32. Multiple Choice Questions UNIT 10 – STRATEGIC ORGANIZATION STRUCTURE Sheela hired a consulting firm to conduct a market comparison of staff salaries before she decided about promotions. Soon after, a rumor started circulating through the organization that a specific group of professionals were going to have their salaries cut as a result of comparison. Staff members did not ask their managers. The rumor spread like wild fire. Immediately, Sheela called an emergency staff meeting and addressed the issue by providing with the facts and gave an opportunity to the employees to speak. What do you call this kind of communication in an organization? a. Formal communication b. The Grapevine c. Upward communication d. Downward communication e. Cross communication When DuPont manufactured gunpowder, individuals involved in the process got specialized. The process resulted in developing separate departments for manufacturing, sales and R&D. The functional heads were reporting directly to the Company President or CEO. To coordinate, sales manager in New York had to interact with other sales managers than with the manager of New York-based plants. Coordination between the plant manager and the sales manager had to occur through the corporate head office. Name the structure followed by DuPont in the case. a. Functional structure b. Divisional structure c. Matrix structure d. Hybrid structure e. SBUs In case no. 92, which of the following factors benefit the individuals? a. Coordination b. Monotony c. Specialization d. a & c e. a & b In continuation to case no.92, once DuPont entered the diversified range of products, its coordination requirements got changed. The production manager of DuPont’s main Dulux paint plant started coordinating with the sales manager for paints and the other functional heads responsible for paints only, than with any other functional head. To facilitate coordination, DuPont changed its structure to ____. a. Geographical structure b. Matrix structure c. Divisional structure d. Functional structure e. None of the above In case no.94, while the divisional structure (product division) promoted coordination, it increases the problem of _______at the top management level. a. Control b. Delegate c. Organize d. Plan e. Review A large conglomerate organization is composed of 90 divisions such as ConAgro, ConFood etc. The chief executive officer faced difficulty in managing the efforts for implementing the strategies. He then decided to divide the many divisions into individual units that has a distinct mission, strategies and objectives and named them as i) Food Service (restaurants) 2) Retail (grocery stores) and 3) Agricultural products. The CEO transformed its structure from divisional to ______________. 19
  • 33. Principles of Management a. Matrix structure b. Functional structure c. Geographical structure d. Hybrid structure e. Strategic Business Unit In case no. 96, each individual business unit named specifically act as a distinct___________. a. Business strategy b. Business entity c. Business Plan d. Business decision e. Business group Philips Electronics was founded in 1892. Philips first focused only on the production of light bulbs. Later it broadened its product lines across different countries. The formal corporate-level structure was built based on the geography and product lines. With growing market changes, Philips realized the need to restructure to cope with the changing global demands. It established national organizations (NOs) with product divisions (PDs), and operated successfully through a network. The network was managed by a number of coordinating committees. These committees helped in resolving the conflicts. What do you call the organizational structure followed by Philips? a. Matrix structure b. Customer division c. Product division d. Geographic division e. Hybrid structure The coordinating committees through networks helped the individuals across the organization to work as ________. a. Unit b. Team c. Panel d. An association e. Entity HP's operations are organized into seven business segments; They are Enterprise Storage, Servers and Networking ("ESSN"), HP Networking, a combination of HP ProCurve and 3Com, (now part of ESSN), HP Enterprise Services ("HPES"), HP Enterprise Security Services ("HPESS"), HP Software Division, HP Technology Services (HPTS), Personal Systems Group ("PSG"), Imaging and Printing Group ("IPG"), HP Financial Services ("HPFS"), and Corporate Investments. Each business segment is organized based on the activities or functions of an organization. What do you call the type of structure adopted by HP? a. Functional structure b. Divisional structure c. Matrix structure d. Hybrid structure None of the above 20
  • 34. Multiple Choice Questions UNIT 11 – EFFECTIVE ORGANIZING AND ORGANIZATIONAL CULTURE Kshitij owns a chemical supplies company. Currently he has a team of 30 members working for his company in different positions. Kshitij is planning to invest the profits he earned in further expansion of business by starting new branches. He started estimating the future personnel needs and the type of proposals for training, if any for expansion. These activities can make an organization effective by_____________. a. Listing Prerequisites b. Avoiding mistakes c. Avoiding organizational inflexibility d. Avoiding conflict by clarification In mobile phone industry, efficient logistics processes and manufacturing capabilities are considered as important success factors. Nokia was indirectly underestimating its own existence in spite of significant process. It deeply failed to consider adaptation, culture, economic environment, creative destruction, leadership and sustainability. Nokia realized its core competency only after forming alliance with Microsoft, when it has all the capabilities. What factor could be the reason for Nokia’s failure in the changing market environment? a. Organizational flexibility b. Organizational conflict c. Organizational relationship d. Organizational culture e. Organizational inflexibility In case no.102, Nokia could sustain through________________. a. Reorganization b. Reengineering c. Reinvention d. Recollection e. Remodification Meenal is a General Manager for a tool manufacturing company. She instructed her sales team to directly communicate with the development team to reduce the time delays in modifying its designs as per customer’s requirement. As the company expanded, many functional teams emerged and some of the employees were reporting to different managers. These changes created chaos and confusion in reporting systemdue to absence of concerned manager’s instructions. The organization could come out of chaos with the help of a definite _______________. a. Organizational chart b. Functional chart c. Organizational culture d. Functional model e. Organizational model Southwest Airlines CEO Herb Kelleher promotes informality and fun at workplace. His practice of acknowledging births, marriages and deaths by sending notes and cards shows how employees are valued. In response, the staff encourages at check-in and thus the time consumed for checking takes less than half the industry average. These characteristics derived from its founders depict Southwest Airline’s__________. a. Social culture b. Organizational structure c. Organizational culture d. Organizational rules e. Organizational programs Wal-Mart founder Sam Walton believed that giving respect to staff builds their self confidence. Walton addresses his staff by their first name and always supports change to sustain in the competitive environment. Which characteristic of organizational culture did Walton promote to maintain the competitive edge? a. Promoting dominant and stable values b. Being distinctive c. Shaping philosophy and rules d. Leading to common behavioral aspects Strengthening through visibility 21
  • 35. Principles of Management The HR manager of Corning, Inc., a manufacturing firm wanted to improve productivity. He focused on reducing the training period and voluntary turnover of new employees. Accordingly the orientation program was designed to foster understanding about the company’s values and to build a positive attitude towards the company. As a result newcomer turnover was reduced by 69%. The example of Corning illustrates ____in the socialization process. a. Step i b. Step iii c. Step vi d. Step vii e. Step v In 1983, Oprah Gail Winfrey got her dream job of hosting a talk-show and soon became one of the most famous and most professional people in the entertainment industry. In 1988, she started the company Harpo Studios. Her television career helped the business to grow by giving her a huge advertising. The company has more than 250 employees now and is growing every year. She also co-founded oxygen media that helped in attracting more than 50 million viewers. Because of the skills she possesses, she has succeeded as _________________. a. An intrapreneur b. An entrepreneur c. An innovator d. A manager e. A leader W.L Gore of Gore-Tex fabric gives his employees 10% of their work day to spend on creativity and personal projects. One employee, Dave Myers, discovered that one of their products, ePTFE(coating for push-pull cables) could be used to coat guitar strings. The coated strings were more comfortable and durable than conventional guitar strings. W.L. Gore launched those strings with a brand name ELIXIR Strings which are now the leading guitar strings sold in the market. W.L. Gore stood as an example for ______________. a. Intrapreneur b. Entrepreneurship c. Executive d. CEO e. Chief Executive Patrick Naughton, a developer in Sun was planning to leave the company in 1995 believing that their management was missing out the demands of PC consumer market. Then he was convinced to continue and develop a group dedicated to the consumer market. Later, this group where James Gosling was a member created a program called object-oriented programming language. It was initially called Oak and later renamed as Java. The above case illustrates that it is the manager’s responsibility to build ___________ that facilitates the achievement of group goals. a. Energy b. Enthusiasm c. Environment d. Ethics Empowerment 22
  • 36. Multiple Choice Questions UNIT 12 – HUMAN RESOURCE MANAGEMENT AND STAFFING Rahul is working as HR manager in a Steel company where the number of current workers is 300. He received a mail from head office to plan for the expansion of production capacity by recruiting skilled workers and a competent supervisor to manage. Keeping the strategic plan of the company in view, Rahul started estimating the size and composition of the future work force. He started preparing the list of skills, competencies and experiences they need to possess. What do you call the process where Rahul is currently engaged in? a. Forecasting manpower supply b. Forecasting manpower demand c. Human resource planning d. Human resource actions e. Human resource management Continuing the case no.111, Rahul first planned to check the current computerized database for the availability of competent and able employee who could take up the position of supervisor. What do you call this means of assessment in human resource planning ? a. Skills inventory b. Succession planning c. Replacement planning d. Human inventory e. Management inventory Continuing the case no.112, Rahul’s efforts to trace the competent individual from the current employees went futile. He then decided to give an advertisement in the newspaper and electronic media. Recruiting people through these resources is called ____________. a. Internal labor supply b. Global labor supply c. External labor supply d. Selection e. Placement After completing Engineering graduation, Meenal went to attend an interview in a software company. She completed all the rounds of interview and at last her name got displayed in the final list. She was asked to report the next Monday. On the day of joining, she is given a list of documents to sign an agreement where all the conditions of organization such as working hours, lunch timing, facilities available and the promotions list including submitting a bond for one lakh rupees in case of violating the agreement of job for two years. In the above case, what are the factors that the organization is taking care of at the time of staffing? a. Personnel policies b. Climate in the organization c. Appraisal system d. All the above e. a & c only Apple Inc., an American computer technology company is known for its innovation. It sells personal computer products and a range of hardware products. In order to survive and react to emerging market demand, it reviews business documents, interprets observations, and uses surveys to measure what is needed by most of the employees within the organization. Based on the results, it effectively implements programs to empower employees and improve their performance. What exactly is Apple Inc. planning to do? a. Staffing b. Performance appraisal c. Compensation d. Layoffs e. Training and development In 2010, Google was facing problems with competitors like Facebook who were attracting employees with stock options. To address the poaching problem, Google transferred a part of employee bonus into the basic salary. It also granted 10% salary increase and offered $1000 bonus in the form of cash. All these initiatives are a part of "competitive _____________." 23
  • 37. Principles of Management a. Training plans b. Compensation plans c. Appraisal plans d. Promotion plans e. Demotion plans Software companies today are approaching educational institutes such as universities, university affiliated colleges and business colleges to recruit potential employees for first level positions. What do you call this type of recruitment? a. External recruitment b. Internal recruitment c. Vertical recruitment d. Horizontal recruitment e. Recruitment by referral An education institution was offering graduation programs in management sciences. Being successful it planned to offer post graduation programs. The top management started listing out the number of subjects in each program to be offered and the number of available faculty for each subject. Then it decided to appoint additional faculty members and started describing the job profiles of the faculty members with detailed specifications. Then it gave a notification in the newspaper and through electronic media for the requirement of faculty members. What is this process called in an organizational environment? a. Selection process b. Production process c. Operation process d. Recruitment process e. Transaction process After qualifying a written test for the post of assistant production manager and being successful in personal interview, Sohail was asked to attend a third round of interview at the production plant. The interview would be headed by the production manager to whomSohail is supposed to report on final selection. These type of interviews are called as a. Personal interview b. Informal interview c. Comprehensive interview d. Exit interview e. Stress interview Standard Chartered Bank selects management trainees from premium B- schools and conducts training program for about six months. During the training period, the new recruiters are given an opportunity to visit several bank divisions and meet the bank’s business heads. It is an effort made to enable the new recruits to get an overall view of the internal operations of bank. Further, team building sessions are also conducted for two days. Before exactly starting to work on the job, the new recruits are given a brief description about the job. This process of introduction of new employee to the organization is called ________________. a. Maximization b. Minimization c. Centralization d. Decentralization Socialization 24
  • 38. Multiple Choice Questions UNIT 13 – PERFORMANCE APPRAISAL AND CAREER STRATEGY McKinsey is a leading strategy consulting firm. It has a HR policy where, managers evaluate their employees on completion of each consulting assignment in addition to the annual performance evaluation. As such, consultants are given around 20 mini-evaluations in one year. This is done to match with the needs of the organization and the development needs of the employee. Identify the objective of appraisals depicted in the case. a. Identification of potential b. Promotion decisions c. Compensation administration d. Validation of selection procedures e. Performance improvement The process followed in the case no.121 is referent to which form of appraisal? a. Informal appraisal b. Formal appraisal c. Standard appraisal d. Systematic appraisal e. Official appraisal Tata Motors incorporated quarterly appraisals in to its performance appraisal system recently. The new system facilitates employees to select their own Key Result Areas (KRAs) for each quarter and gives an opportunity to review their performance based on these parameters. These types of appraisals are called _________. a. Standard appraisal b. Formal appraisal c. Informal appraisal d. Systematic appraisal e. Official appraisal In Tata Motors, major revenue is generated by sales department. As such, sales people are given increments in cash and promotions based on their performance. Rahul, who was not performing up to the satisfaction, was given training. Consequently, his performance improved and he was also given cash benefits. Name the quality control technique used by Tata Motors in the case, to motivate Rahul. a. Standards b. Information c. Corrective action d. Performance action e. Management action Frito lay products include Lays, Sun chips and Doritos. It has more than 50,000 employees working out of which one third are route sales representatives (RSR). Their job is to negotiate and put products on store shelves. Their performance was measured through sales figures as the job was paid on commission basis. The management observed that profitability goals were not met becaus e of low productivity and high RSR turnover. Soon, the management detected that more than sales figures, sales skills had the greatest impact on the sales and introduced other performance behavior based measures to evaluate RSR such as sales task, driving and delivery tasks and merchandising tasks. Name the latest performance management adopted by Frito lay. a. Graphic rating method b. Management by objectives (MBO) c. 360 degree appraisal d. Comprehensive appraisal e. Behaviorally Anchored Rating Scales (BARS) In a glass manufacturing company, management followed conventional output based appraisal system. In performance appraisal meeting, the production manager identified that most of the knowledgeable employees were not satisfied by the system. He then proposed a change in the appraisal system where quality of work, job knowledge, dependability, punctuality and attendance are considered in addition to output for performance appraisals. Identify the assessment method proposed by the production manager in the case. 25
  • 39. Principles of Management a. Graphic Rating Method b. Behaviorally Anchored Rating Method c. Forced Ranking Method d. Critical Incident Method The Narrative Method Production manager in a bearing company got instructions to produce 100 tonnes within three months as against the actual capacity of 80 tonnes. The manager put all his efforts to meet the target by giving required resources and instruction to the workers and planned to review the progress every month. By the end of second month, they could produce 70 tonnes. Unfortunately, message received by the manager from the Head office stated that the order has been replaced for only 80 tons which is their normal target. The efforts of the manager went unnoticed by the management during performance appraisal. Which factor of MBO implementation played the role in performance management? Strengths of appraisal against verifiable objectives Opportunities of appraisal against verifiable objectives Weaknesses ofappraisal against verifiable objectives Threats of appraisal against verifiable objectives None of the above Pratheek, a good performer as a system administrator was recommended for a training program in the latest java development courses. But his interest was always to learn the new Hadoop technology yet to be popular in the market. He tried to express his view to the management about his concern but things did not work and he had to undertake the training program. Later, under the real project, his performance was not up to the expectations. Where lays the reason for Pratheek’s low performance in the case? a. Implementation of a career plan b. Monitoring progress Analysis of persona strengths and weaknesses Consistency testing and strategic choices Analysis of environment threats and opportunities Tesco follows an appraisal system called 'talent planning'. This practice promotes people to work with commitment in the organization. Individuals are allowed to apply for promotions in the areas of their career interest through annual appraisal scheme. Their manager helps them in acquiring the technical skills and behaviors required for these roles by conducting training sessions. This is how Tesco achieves its business objectives and facilitates its employees to achieve their personal and career objectives. Name the activity taken up by the Tesco managers under career strategy formulation. Preparation of a personalprofile Analysis of environment threats and opportunities Development of strategic career alternatives d. Monitoring progress e. Implementation of a career plan Team leader in a software company was planning to conduct a training session for his team members. He started comparing the current competencies of his team members against the organizational requirements and market trends to find the performance gaps. For the purpose, he also checked their completed projects for the identification of potential training needs. But all these efforts were time consuming, he then decided to list out such gaps through performance appraisals from the next time. In which part of the career strategy do you realize this concept? a. Monitoring progress b. Monitoring training needs Development of short-range career objectives and action plans Development of strategic career alternatives Development of long-range personaland professionalgoals 26
  • 40. Multiple Choice Questions UNIT 14 – ORGANIZATIONAL CHANGE AND ORGANIZATION DEVELOPMENT In 2010, IBM introduced Accessible Workplace Connection (AWC), a process of giving special facilities to handicapped people to complete their work. This assistive technology is also called as a “one-stop-shop”. It allows interaction between employee s and the in charges of IBM teams providing accommodations. AWC are established to provide continuous support by giving reasonable and effective accommodations to eliminate process confusion. Name the factor that forced change in the workplace environment in IBM. a. Technology b. Economic factors c. Competition d. Nature of the workforce e. Political factors An organization offering business analysis services decided to start a cost reduction program. It searched for potential chances in the company and found most of the employees using individual printers connected to their computer in the office. When approached IT department for an alternative solution, they proposed replacing the individual printers with networked printers and placing them at convenient work stations. This change was estimated to save $2 million annually. The proposal to that effect was signed by the senior management and individual printers were slowly being replaced. Meanwhile, majority of employees complained great inconvenience and reported to the top management due to which the program was postponed losing the significant opportunity costs. What is the cause for losing opportunity costs? a. Organizational resistance b. Individual resistance c. Group resistance d. Customer resistance e. Stakeholder resistance An electronic product manufacturing company wanted to eliminate unnecessary processes and improve the use of materials by employing lean manufacturing system. In the process, it had to change the procedures for ordering and had to lay off some of its employees. In a way the change has affected the entire system of operations in the company leading to further cost and design assessment. The organization temporarily put off the plans for the support from all the departments. Which source of organizational resistance is affected in the plan? a. Limited focus b. Threat to expertise c. Threat to established power relationships d. Threat to established resource allocation e. Group inertia In case no.132, the top management gave instructions on how to motivate employees to accept the change. Accordingly, middle level managers conducted meetings department wise and explained the benefits of using printers through networks and the concept of cost saving. They also explained how these benefits shall reap fruits for employees. Identify the factor that helped the managers to overcome resistance. a. Supervision and control b. Facilitation and support c. Negotiation and agreement d. Manipulation and co-optation e. Education and communication In case no.133, Top management called for a meeting with all the heads of departments and requested to participate in the discussion on the change consequences. The top management tried to clear the fears about the change and asked them to come up with plans on how to implement the change gradually with the consent of employees. Name the factor that helped the management to overcome the resistance of change. 27